83603384-mcq

March 18, 2018 | Author: vipul51190 | Category: Human Tooth, Tooth Enamel, Dentures, Tooth, Hepatitis B


Comments



Description

MCQ Questions Preliminary examinationAll of these questions are based on what people remembered after exams SO IT CAN NOT BE RELIED ON 100% but it is the only way to get an idea about the subjects, matters and topics you would be asked about. Please add whatever you can after the exam ends and keep this sample in the hands of any who is sitting the exam. By the way it took me about 4 months to get this organised the way it is now. THANKS to all who have contributed to this and to all who will. Hadi 1. For lower premolars, the purpose of inclining the handpiece lingually is to, A. Avoid buccal pulp horn B. Avoid lingual pulp horn C. Remove unsupported enamel D. Conserve lingual dentine 2. For an amalgam Restoration of weakened cusp you should, A. reduce cusp by 2mm on a flat base for more resistance B. reduce cusp by 2mm following the outline of the cusp C. reduce 2mm for retention form 3. Before filling a class V abrasion cavity with GIC you should, A. Clean with pumice, rubber cup, water and weak acid B. Dry the cavity thoroughly before doing anything C. Acid itch cavity then dry thoroughly 4. Which of the following statement about the defective margins of amalgam restoration is true? A. The larger the breakdown, the greater the chance of decay. 5. The retention Pin in an amalgam restoration should be placed, A. Parallel to the outer wall B. Parallel to the long axis of tooth 6. The most common cause of failure of the IDN “Inferior Dental Nerve” block is, A. Injecting too low B. Injecting too high 7. Which one of the following are not used in water fluoridation: A. SnF2 B. 1.23% APF C. H2SiF2 D. CaSiF2 E. 8. 8% Stannous fluoride The best way to clean cavity before the placement of GIC is, A. H2O2 B. Phosphoric Acid C. Polyacrylic acid 9. The most mineralised part of dentine is, A. Peritubular dentine 10. A 45 year old patient awoke with swollen face, puffiness around the eyes, and oedema of the upper lip with redness and dryness. When he went to bed he had the swelling, pain or dental complaints. Examination shows several deep silicate restorations in the anterior teeth but examination is negative for caries, thermal tests, percussion, palpation, pain, and periapical area of rarefaction. The patient’s temperature is normal. The day before he had a series of gastrointestinal x-rays at the local hospital and was given a clean bill of health. The condition is: A. Acute periapical abscess B. Angioneurotic oedema C. Infectious mononucleosis D. Acute maxillary sinusitis E. Acute apical periodontitis 11. Internal resorption is, A. Radiolucency over unaltered canal B. Usually in a response to trauma C. Radiopacity over unaltered canal 12. On replantation of an avulsed tooth could see, A. Surface resorption, external resorption B. Internal resorption C. Inflammatory resorption D. Replacement resorption E. A, C and D F. All of the above 13. The percentage of total dentine surface dentinal tubules make in 0.5mm away from pulp is, A. 20% B. 50% 14. The junction between primary and secondary dentine is, A. A reversal line B. Sharp curvature C. A resting line D. A reduction in the number of tubules 15. What is the correct sequence of events A. Differentiation of odontoblast, elongation of enamel epithelium, dentine formation then enamel formation. B. Differentiation of odontoblast, dentine formation then enamel formation, elongation of enamel epithelium. C. Elongation of enamel epithelium, differentiation of odontoblast, dentine formation then enamel formation. 16. What is the sequence from superficial to the deepest in dentine caries? A. Zone of bacterial penetration, demineralisation, sclerosis, reparative dentine B. Zone of bacterial penetration, reparative dentine, demineralisation, sclerosis. C. Zone of bacterial penetration, sclerosis, reparative dentine, demineralisation. 17. The nerve supply of the pulp is composed of which type of nerve fibres? A. Afferent & sympathetic 18. In which direction does the palatal root of the upper first molar usually curve towards? A. Facial / buccal/ B. Lingual C. Mesial D. Distal 19. What is the common appearance of vertical tooth fracture? A. Perio abscess like appearance B. Displacement of fragments 20. Which of the following would be ONE possible indication for indirect pulp capping? A. Where any further excavation of dentine would result in pulp exposure. B. Removal of caries has exposed the pulp C. When carious lesion has just penetrated DEJ 21. Following trauma to tooth, the next day there was no response to pulp tests you should? A. Review again later B. Start endodontic treatment C. Extraction of tooth 22. Extreme susceptibility to pain B. There is a profound amnesic action and no side affects D.1ml of blood from Hepatitis B carrier is less infective than 0. What is the main purpose of performing pulp test on a recently traumatised tooth? A. Which is NOT TRUE in relation to the prescription of 5mg or 10mg of diazepam for sedation? A. what would you recommend as the BEST method to verify that sterilization has occurred:** A. The answered questionnaire shows that she is suffering from severe cirrhosis. Obtain accurate indication about pulp vitality 23. Decalcification of dentine B. Use colour change tape daily and spore test monthly E. 0. Use colour change tape in each load and spore tests weekly 27. A 65 year old woman arrived for dental therapy. Cleaning debris from root canal 24. 0. Use spore test daily B. Active metabolites can give a level of sedation up to 8 hours post operatively E. Use indicator strips daily and spore test weekly D. Tendency towards prolonged haemorrhage C. The problem that can be anticipated in the routine dental therapy is: A.1ml of blood from Hepatitis B carrier is more infective than 0. Which of the following is TRUE in regards to high risk patient? A. Patient commonly complain of post operative headache B. Use indicator strips in each load and colour change tape on each package C. Your employer in an attempt to update office sterilization procedures. Level of virus are similar in the blood and saliva of HIV patient D.1ml of blood from HIV patient C. What is the main function of EDTA in endodontics? A. Level of virus in the saliva is not significant for Hepatitis B patient E. Obtain baseline response B. As Benzodiazepine the action can be reversed with Flumazepil 25. Recurring oral infection . The presence of Hepatitis B core Antigen in the blood means that active disease is not present 26.1ml of blood from HIV patient B. An acceptable level of anxiolytic action is obtained when the drug is given one hour preoperatively C. Elevated temperature and nausea B. Exteriorizing the cyst through the buccal bone and mucosa C. Signs and symptoms that commonly suggest cardiac failure in a patient being assessed for oral surgery are. Ankle oedema and dyspnoea D. A. Pallor and tremor 31. Trigeminal neuralgia D. Making a mucoperiosteal flap and removing the cyst through an opening made in the alveolar bone. A persistent oroantral fistula for a 12 weeks period following the extraction of a maxillary first permanent molar is best treated by. Further review and reassurance since it will most probably heal spontaneously B. 32. Routine orthograde endodontic treatment followed by observation. Patient received heavy blow to the right body of the mandible sustaining a fracture there. sub-condylar region 30. which method of treatment would you consider?** A. You should suspect a second fracture is most likely to be present in. Loss of sensation in the lower lip may be produced by. Left sub-condylar region D. E. Erythema and pain E. Fracture in the mandible first molar region E. Left body of the mandible C. Excision of the fistula and surgical closure . Palpitations and malaise C. Increased tendency to syncope Difficulty in achieving adequate local anaesthesia 28. A. D. Making a mucoperiosteal flap and removing the cyst through an opening made in the alveolar bone. Symphysis region B. Right sub-condylar region E. Curettage and dressing of the defect D.D. A cyst at the apex of an upper central incisor measuring 1 cm in diameter is visualized in radiograph and confirmed by aspiration biopsy. Ludwig’s angina 29. E. A. Extraction of the central incisor and retrieving the cyst through the socket B. Antibiotic therapy and nasal decongestants C. Bell’s palsy B. Traumatic bone cyst C. followed by tooth removal. A. followed by endodontic treatment. which of the following is not one of these factors? A. Presence of lymphocytes C. Collection of neutrophils D. The most significant finding in clinical evaluation of parotid mass may be accompanying. The use of vasoconstrictors in local anaesthetics will increase the chances of infection. Maxillary antral wash out and nasal antrostomy. Suppuration is mainly the result of the combined action of four factors. Xerostomia 34. Nodular consistency C. A. Serotonins E.E. 33. which of the following describes this role? A. As far as surgical removal of wisdom teeth is concerned which of the following is true? ** A. Autolysis by proteolytic enzymes . Prophylactic prescription of dexamethasone will dramatically reduces post operative swelling D. Enkephalins 36. Prostaglandins D. 35. Facial paralysis D. Raising a lingual flap will increases the incidence of neurapraxia but will reduce the incidence of neurotmesis with respect to the lingual nerve C. Endogenous morphine like substance which can control pain is known as. They supply fibrin stabilizing factors E. Slow progressive enlargement E. Prophylactic prescription of antibiotic reduces dramatically the chances of infection B. They convert fibrinogen to fibrin B. They agglutinate and plug small. Inferior dental nerve injury is unlikely since the nerve passes medial to the wisdom tooth root E. Lympha adenopathy B. Bradykinins B. Necrosis B. Platelets play an important role in haemostasis. They supply proconvertin for thromboplastin activation 37. Peptides C. ruptured vessels C.** A. Accumulation of tissue fluid E. They initiate fibrinolysis in thrombosis D. The pain usually last for few seconds up to a minute in the early stages of the disease B.38. Infection with pneumocystic carinii D. The pain is usually unilateral C. Kaposi sarcoma on the palate B. Furrows outlined the dorsal surface radiating out from a central groove in the centre of the tongue C. Patient characteristically have sites on the skin that when stimulated precipitate an attack of pain D. Herpes simplex infections B. Which of the following have a tendency to recur if not treated? . Always associated with trauma to the lateral side of the tongue E. Reduction in white cells count E. Pemphigus vulgaris C. Irregularly outlined erythematous area of hyper trophic fungiform E. Which of the following lesions CANNOT BE classified as an intra-epithelial lesion?** A. Associated with HIV virus infection and is commonly seen on the lateral side of the tongue C. Benign migratory glossitis or Geographic Tongue. Usually caused by Candida species D. It is a paroxysmal in nature and may respond to the treatment with Carbamazepine 41. Which one of the following is true about oral hairy leukoplakia? A. foot and mouth disease 39. manifests itself in the oral cavity as. A. Grooves (fissures) radiating from a central fissure G. Irregular area in the midline of the tongue 42. Associated with HIV virus infection and is commonly seen on the dorsal of the tongue B. Which of the following is NOT CHARACTERISTIC of trigeminal neuralgia?** A. In regards to HIV infection. B cell lymphoma 40. An attack of pain is usually preceded by sweating in the region of the forehead E. Hand. Reduced haemoglobin C. Always associated with pernicious anaemia 43. Lichen planus E. Irregularly outlined areas of hyperkeratosis of the dorsal surface of the tongue B. Herpangina D. Loss (atrophy) of filiform papillae in multiple irregularly outlined areas D. A fibrinous exudate on the dorsal surface F. which of the following is the earliest finding? A. Thermodynamic action B. Type I allergic reaction B. A. Lipoma C. Anterior dorsal surface C. Fibrous epulis D. Posterior dorsal surface D. Trichloroacetic acid. Lateral border anteriorly B. Giant cell granuloma B. Basal cell carcinoma is characterised by. Radiation resistance F. Haematoma E.A. Can not metastasise to the bone 45. its mechanism of action is. Rapid growth and metastasis B. A patient presents complaining of a stomach upset 48 hours after starting a course of antibiotic for oral infection. Lateral border posteriorly E. No preferred location 46. Uterine neoplasia D. Pyloric stenosis 47. Local cutaneous invasion C. Poor prognosis E. A. Neutralization 48. Nervous disorder C. Type IV hypersensitivity reaction E. a strong acid. Side effect of the drug D. A. this is an example of. Osmotic pressure D. Thromboembolism disorder . Hypotension B. Inability to invade bone D. Activation of tissue enzymes C. Carcinoma of the tongue has a predilection for which of the following sites?** A. Protein precipitation PPT E. has been used by dentists for chemical cautery of hypertrophic tissue and aphthous ulcers. Which of the following adverse reaction of oral contraceptives is the most common and the most serious A. Hepatotoxicity C. Pulp polyps 44. Thrombocytopenic purpura .E. Synthesis of prostacyclin and prevents platelet aggregation C. Myelogenous leukaemia B. Platelets=250000. Thrombin and prevents formation of the fibrin network E. spontaneous bleeding from the gingiva and complains of weakness and anorexia. The disease that you would immediately suggest is. Hepatitis B carriage in institutionalised patients 52. Thyrotoxicosis C. Adrenal insufficiency 51. Sever caries but minimal periodontal disease C. Which one of the following preoperative haematological analyses may reflect this disease: A. Decreased resistance to infection 49. Typical features of Down’s syndrome (Mongolism) do not include: A. A 22 year old woman has acute gingival hypertrophy. Synthesis of thromboxane A2 and prevents platelet aggregation B. Eosinophils=0% The most likely diagnosis is: A. Infectious mononucleosis /glandular fever/ C. Neutrophils=90%. A multiple immunodeficiencies B. Susceptibility to infections D. Increase the absorption of vitamin K and prevents synthesis of blood clotting factors 50. Thromboplastin generation time 53. Hyperparathyroidism D. Osteoporosis E. Lymphocytes=9%. A. A patient who recently had a calculus removed from the kidney presented with radiolucent area in the left maxilla with clinical evidence of swelling. Diabetes B. Bleeding time and factor VIII level B. The patient whom you are about to treat. A patient who has been taking quantities of aspirin might show increased post operative bleeding because aspirin inhibits:** A. WBC=100000. Bleeding time and factor X level D. Platelet count E. Her blood analysis was as follows: HB=12gm. Bleeding time and factor IX level C. Synthesis of prostaglandin and prevents production of blood platelets D. Monocytes=1%. states that he has Von Willebrand’s disease. Multiple missing teeth and malocclusion E. The tonsillar lymph node is situated at the level of. Gingivitis of local aetiological origin Pernicious anaemia /Vitamin B12 deficiency/ 54.D. Exposure of the patient to ionising radiation when taking a radiograph is NOT REDUCED by: A.0s C. Triple the time . Double D. No effect B. Silver bromide in gelatine D. Decreasing the kilovoltage KvP 56. Silver nitrate in gelatine E. Quarter E. You wish to purchase a dental X ray machine and have the choice between 60kVp and 70kVp machines. A. Jugulodigastric crossing D. The use of fast film B.5s B. The emulsion contains particles of. 0. The use of an open and lead lined cone E.25s E. C6 vertebrae C. 0. Silver nitrate crystal B. What would be the exposure for the same situation with 400mm target to film distance? A. Clavicle E. The addition of filtration C. the exposure time was 0. 2. Half the time C. E. Angle of the mandible B. With single change from 60kVp to 70kVp what would the approximate affects on exposure time? A. Collimation of the beam D. X-ray films have an emulsion on one or both side of a support material.0s D.125s 58. The inverse Square Law is concerned with intensity of radiation using type D film of 200mm target to film distance. 0. 1. A. Potassium bromide in gelatine 57. Metallic silver in gelatine C.25s. Jugulo-omohyoid crossing 55. Mandibular incisors D. because of lateral constriction. 0. presents for treatment three hours after injury. Yes. 2 metres E. When primary molars are prepared for stainless steel crowns should the depth for reduction of the proximal surface be similar to the depth of the buccal and lingual surfaces? A. No. 1. Maxillary and mandibular canines C. Mandibular molars 64. Pulpectomy and apexification 63.59. Zinc oxide and eugenol cement D. Pulpotomy using formocresol D. No.5 metres D. all undercuts are uniformly removed so that the steel crown can be seated E. 3 metres 60. A. No. reduction of all wall is similar for best retention B. Maxillary molars B. Maxillary incisors E. proximal reduction is greater to allow the crown to pass the contact area C. the lingual surface needs greatest reduction 62. Polycarboxylate cement 61. Zn phosphate cement and formcresol combination paste B. Remove the surface 1-2 mm of pulp tissue and place calcium hydroxide B. Which of the following should be considered? A. the operator should stand out of the primary x ray beam and a distance from the patient’s head of at LEAST: A. Which of the following anomalies occurs during the initiation and proliferation stages of tooth development . Yes. the buccal surfaces has the greatest reduction to remove the cervical bulge D. Quick setting hydroxide cement C. Place calcium hydroxide directly on the exposed pulp C.5 metres B. The obturating material of choice for primary teeth following complete pulpectomy is. Which primary teeth are LEAST affected with the nursing bottle syndrome? A. Pulpectomy and immediate root filling E. 1 metre C. When no radiation shield is available. 8 years old child who has sustained a fracture of maxillary permanent central incisor in which 2mm of the pulp is exposed. Gutta-percha E. A health 6 year old child presents with carious maxillary second primary molar with a necrotic pulp. Have adequate overbite C. Impaction of 15 and 25 between first premolars and first molars D. Use posterior capping E. Overlapping of lower incisors B. morphodifferentiation. morphodifferentiation. morphodifferentiation. histodifferentiation. Ankylosis 65. initiation. Proliferation. Which is the right sequence of the histological stages of tooth development? A. histodifferentiation. Extraction B. mineralization D. The lamina dura seen on periapical radiograph as: A. Amelogenesis imperfecta B. Treat during growth D. Dentinogenesis imperfecta C. Pulpectomy E. Palatal displacement of upper canines C. Which treatment would be preferred? A. Cribriform plate of bone making the tooth socket C. it is essential to: A. mineralization C. mineralization B. Dense crestal bone consistent with a healthy periodontal status D. Rotation of 16 and 26 69. Initiation. proliferation. Indirect pulp treatment C. proliferation. Increase vertical dimension 68. assuming no primary teeth has been lost prematurely? A. histodifferentiation. Which of the following are typical consequence of dental crowding. Oligodontia E.A. mineralization 66. Antibiotic coverage 67. histodifferentiation. To produce a stable correction of an upper labial segment in lingual crossbite. Pulpotomy D. B. Usual radiolucency between tooth root and surrounding bone as a thin white line. Pattern of radiopaque lines in supporting alveolar bone . Initiation. Enamel hypoplasia D. Use fixed appliances B. Proliferation. morphodifferentiation. Mesial tipping of 16 and 26 E. Hyper mobility indicates that the tooth supporting structure have been weakened D. Evaluate the soft tissues 10 to 14 days later. Is frequently caused by lichen planus D. It is caused by hormononal imbalance B. The treatment of Localised Juvenile Periodontitis is frequently supplemented with tetracycline because flora involved is predominant: A. Spirochaetes and eikenella corrodes C. The most accurate way to evaluate the effectiveness of root planning is by: A. Which of the following organisms are pathognomonic of acute necrotic ulcerative gingivitis? A. Is seen only at or after menopause C. Activate a curette against root surface and listen for a high pitched sound which indicates a smooth. Polymorphs and lymphocytes D. Reliance on radiograph is essential 72. Is related to nutritional disturbance 73. Aerobic B. 75.70. which of the following statement is true: A. Use air for visual inspection C. Heavy pressure must sometimes be used to test mobility B. Porphyromonas gingivalis and prevotella intermedia 71. Resistant to other antibiotic 74. Spirochaetes and fusobacterium SP B. Is a variant pregnancy gingivitis E. During the periodontal examination each tooth should be tested individually for hyper mobility E. In testing for mobility. Only lateral mobility is significant in diagnosis and treatment of chronic inflammatory periodontal disease C. D. Feel the top of the crestal bone . Inspect the root surface with an instrument for root smoothness B. hard surface. Probe pressure at the sulculus of pocket should not be more than enough to: A. Strictly anaerobic C. Actinobacillus actinomycetes comitans oral capnocytophaga E. Which of the following is true regarding gingivosis (Desquamative gingivitis) A. Evaluate the soft tissue at the end of the appointment for a decrease oedema and bleeding E. Facultative or microaerophilic D. the major reaction is: A. A curette may be inserted to the level of the attached gingiva with minimal trauma to the tissues because of: A. Pre-treat the enamel with 0. Several approaches have been suggested to increase the fixation of professionally applied topical fluoride. Stannous fluoride D. Fluoroapatite 80. Calcium fluoride C. Aids in re-mineralising the root surface D. Tetracycline hydrochloride conditioning of root surface in periodontal surgery is to: A. Prevents post operative infections 78. Provides good tactile sensitivity E. A. D. E. Raise the PH of the fluoride solution C. Of all the factors that increase the resistance of teeth to dental caries THE MOST EFFECTIVE is. When the enamel of the tooth is exposed to preparation containing high concentrations of fluoride. Has a round base B. Has rounded cutting edges D. Increase the exposure time to topical fluoride D. Use NH4F rather than NaF at a lower PH . Sufficient intake of calcium and Vitamin D during the period of enamel mineralization and maturation 79. Increase concentration of fluoride in solutions B. May enhance binding of fibronectin and fibroblast C. Is easy to sharpen C. The intake of fluoride during the period of enamel mineralization and maturation C. Periodic topical fluoride application by dental health care following tooth eruption D. Has two cutting edges 77. Sodium fluoride B.B. Assist the biding of lamina dura E. The general nutrition of a child during the period of tooth formation B. which of the following statements IS INCORRECT regarding increasing the fixation? A.5% phosphoric acid E. C. Sterilise the root surface B. Balance the pressure between fulcrum and grasp Define the location of the apical and the calculus deposit Feel the coronal end of the attached tissues Limit the lateral pressure 76. Cysts D. D. Medial and lateral pterygoid C. What is the purpose of making a record of protrusive relation and what function does it serve after it is made? A. you should use: A. B.Buccal in lower first molar 86. To aid in establishing the occlusal vertical dimension and to adjust the condylar guides of the articulator so that they are equivalent to the condylar paths of the patient. Temporal and lateral pterygoid E. Mesio–Lingual in lower first molars E. Distal pressure and movement D. The main factor controlling a decision to increase the occlusal height of teeth for extensive oral reconstruction is whether. Disto–buccal in lower first molars D. In the inferior alveolar block the needle goes through or close to which muscles: A. Biopsy is least useful in the diagnosis of. To aid in determining the freeway space and to adjust the inclination of the incisal guidance. Rotation C. 85.81. Geographic tongue B. Labial-lingual movement 84. To register the condylar path and to adjust the inclination of the incisal guidance. Temporal and medial pterygoid 83. A. Granuloma E. The extraction of maxillary deciduous molar in 5 years old child. C. the pulp horn most likely to be exposed in the preparation of large cavity in permanent molar tooth is. The inter occlusal distance will be physiologically acceptable after treatment . A. Buccinator and superior constrictor B. Medial pterygoid and superior instructor D. Mesio–Lingual in upper first molars B. Myeloma 82. Mostly towards the apex pressure and some movement B. Aphthous ulcer C. A. Mesio. To register the condylar path and to adjust the condylar guides of the articulator so that they are equivalent to the condylar paths of the patient. Mesio–Buccal in upper first molars C. A. The mandibular arch is relatively narrower than the maxillary arch . There will be sufficient tooth bulk in the abutment teeth for proper retention of the crowns C. In planning and construction of a cast metal partial denture the study cast. Lateral pterygoid C. Mentalis B. provide only limited information about inter ridge distance. Following extraction of the molar teeth** A. Maintaining the clasp arms on all abutment teeth at the ideal degree of tension 89. A. A. Employing bar clasps on all abutment teeth C. You decide to increase the occlusal vertical dimension by 4mm this will necessitate. A new centric relation to be recorded C. The maxillary ridge will get more bone lost from the palatal aspect than the buccal C. facilitate the construction of custom/special trays B. An increase in the rest vertical dimension 91. Clasping at least two teeth for each edentulous area E. Periodontal damage to abutment teeth of partial denture with distal extension can best be avoided by. Jaw relation of an edentulous patient has been established. Temporal 90. A change in the condylar guide settings D. Which of these muscles may affect the borders of mandibular complete denture. can be used as a working cast when duplicating facilities are not available 88. Orbicularis oris D. Levator angulioris E. minimize the need for articulating C. Maintaining tissue support of the distal extension D. A. which is best assessed clinically D. The maxillary cast has been mounted on an articulator without a face bow. The ridge height is lost more from the maxilla than from the mandible B. The aesthetic appearance of the patient will improve sufficiently to warrant the planned reconstruction 87. At least two third of the original alveolar process will remain for adequate periodontal support D. Applying Stressbreakers B. Opening the articulator 4mm B.B. A. The principle muscle responsible for the opening of the mouth is. All deciduous teeth and the first permanent molars D. Which of the following is a major disadvantage to immediate complete therapy. Posterior temporal D. the last secretion of the odontoblast is cementum X C. The remnants of Ameloblast contribute to the primary enamel cuticle B. the mandibular ridge will lose more bone from the lingual aspect than the buccal one. A. some calcified dental tissues are presented. Deciduous teeth only 96. Anterior temporal C. The last secretion of the ameloblast is the acquired of enamel cuticle D. Enamel must contain glycoproteins C. Pulp must contain complement 94. Fructose C. For dental caries to progress in dentine. 92. Trauma to extraction site B. which one of the following statement is correct. All deciduous teeth and permanent central incisors C. Excessive resorption of residual ridge 93. Amylopectin E. All deciduous teeth and all permanent incisors B. Anterior belly of digastric denture . Sucrose D. Compared with the pre-resorption state. Refer to Boucher Microbiology A. A. A. Increased the potential of infection C. Dextrans 95. Mylohyoid B. Diet must contain polysaccharides E. A. Glucose B.D. The dentine must contain soluble collagen B. At birth. The remnants of odontoblast form the primary enamel cuticle 97. Diet must contain simple carbohydrate D. Streptococcus mutans utilise which subtract to form dextran. Impossibility for anterior try in D. All of the above 103. What would not cause an airway obstruction A. Hypercementosis D. To distribute the load between teeth and ridges B. sella… 100. Nasion. Tooth under occlusal trauma shows A. What are the points that determine the facial line in cephalometric points. subnasale. Class III inlay with pins 104. “ The angle of the convex facial line”: A. Pinlay Veneer D. To distribute the load between the clasps and the face end of the saddle C. What is Miller’s theory about A. Which is more retentive form for anterior bridge A. The relevant jaw C. Bone resorption B. It relieves the abutment tooth of occlusal loads that may exceed their physiologic strength 101. Acidogenic micro-organism B. Proteolytic The researcher name is W. Triangulation E.98. The relevant quadrant 99. What is the main purpose of using Stress breakers: A. Laryngeal muscles paralysis B. Airway obstruction D. Class V inlay C. Flexion of the neck C. Extension of the neck . D. Necrosis of the pulp C. Orbital. Miller 102. ¾ partial veneer crown B. Loss of tooth in mixed dentition affects the A. The whole mouth D. pogonion B. Same quadrant B. A child has sustained a traumatic exposure of primary central incisor. Pulpotomy and Ca(OH)2 B. The most appropriate diagnosis and treatment plan in such situation would be:** A. but yet only three permanent first molars are erupted. surgical soft tissues exposure C. B. The prophylactic prescription of antibiotics prior to extraction reduces the incidence. surgical enucleation. A. Idiopathic failure of eruption. which one of the following is true? A. Ankylosis of the molar. Mandible to cranial base B. he presents to you for treatment two days after the injury. best signifies the relationship of. A 10 year old boy presents with small greyish white lesion surrounded by a red halos on the soft palate and tonsillar pillars. A patient with impacted canine. Excessive fibrinolysis is the likely aetiology D. Palatally impacted 107. The MOST probable diagnosis is? A. Oral examination reveals a large gingival bulge in the un-erupted permanent area. Maxilla to cranial base C. Zinc oxide eugenol and alvogyl dressing promote a rapid bone growth 106. Maxilla to mandible D. He has fever and pain in the ear. Purulent exudate must be seen for a diagnosis and irrigation is mandatory E. Which of the following should be considered? A. Mandible to porion E. The SNA angle on cephalogram. small vesicles are found. removal of the first molar to allow the second one to . Pulpectomy (RCT) 110. Direct pulp capping D. where do you expect the impacted canine: A. As far as localised alveolar osteitis is concerned. Dentigerous cyst. 8 years old child presents with all permanent incisors erupted. The incidence in the mandible and maxilla is similar B. Pulpotomy and formocresol C. A panoramic radiograph shows the alveolar emergence of the un-erupted permanent first molar crown and three fourth tooth developments. by moving the X ray tube distally the canine moves distally too.105. Maxilla to Frankfort plane 109. Labially impacted B. C. there are no other radiographic abnormalities. Herpangina 108. Oral examination reveals a large gingival bulge in the un-erupted permanent area. Which of the following laboratory results would be a possible indicator of this? A. C. Locally debride. Patient presents with rapidly progressive root caries on many teeth. Dentigerous cyst. S. Prescribe Metronidazole 100mg B. 12 years old child presents with symptoms of widespread gingivitis with bleeding and general malaise for several weeks. surgical enucleation. Ankylosis of the molar. D. Stimulated salivary secretion rate of 1. there are no other radiographic abnormalities. A. Idiopathic failure of eruption. A panoramic radiograph shows the alveolar emergence of the un-erupted permanent first molar crown and three fourth tooth developments. An increased susceptibility to periodontal disease E. mutans concentration of 105 organism/ml C. Idiopathic failure of eruption.erupt into its place. surgical soft tissues exposure and orthodontic traction. surgical soft tissues exposure C. B. but yet only three permanent first molars are erupted. 111. Give a prophylaxis with ultra sonic scaling . 8 years old child presents with all permanent incisors erupted. surgical soft tissues exposure and orthodontic traction. mutans D. Which of the following is NOT characteristic of Down’s syndrome? A. Ankylosis of the molar. Carcinoma of the lips 114. A plaque sample containing 5% S. surgical soft tissues exposure and luxation of the molar E. A lactobacilli concentration of 105 organism/ml E. Macrodontia D. D. Salivary buffering PH 5. The MOST common carcinoma in the mouth is. Decreased neutrophil function B. 115. Macroglossia C. removal of the first molar to allow the second one to erupt into its place. Congenitally missing teeth 113.5 112. give oral hygiene instruction and prescribe H2O2 mouth wash. Epidermoid carcinoma /Squamous Cell Carcinoma/ B.5ml/min B. Idiopathic failure of eruption. surgical soft tissues exposure and luxation of the molar E. Ankylosis of the molar. The most appropriate diagnosis and treatment plan in such situation would be: A. How would you manage this patient? A. Tooth brushing immediately after meals is most effective because demineralisation has already started C. The Stephan Curve describes an increase in PH during a meal with resultant of demineralisation 119. The BEST treatment for alveolar abscess: A. Foods that require vigorous mastication will increase salivary flow and reduce PH B. Indication of bacterial activity C.D. In developing plaque. Sucrose D. Lactose E. Food that encourage the mastication will increase the number of lymphocytes in saliva and thus reduce decay D. Endodontic 120. Glucose B. Endontic treatment or extraction B. Amylose . Refer for haematological screening Advise for bed rest with supportive and palliative treatment 116. Prevent gingivitis E. Fructose C. Extraction D. Provide a long term improvement in oral hygiene C. What is the affect of office dental prophylaxis of regular six month intervals on children’s oral health? A. Reduce the need for patient cooperation 117. Reduce caries incidence by approximately 30% B. It is common to both animal and human 118. Vigorous mastication will increase plaque PH and lead to reduce of decays E. Antibiotic therapy prevents or stop its formation B. Provide a short term improvement in oral hygiene D. the adhesive polymer produced by streptococcus mutans is synthesis from: A. Incision and drainage alone C. Plaque is considered as an infection because: A. E. Which of the following is true in relation to dental decay? A. There is a possibility of occlusal disharmony 127. Fixed bridge using the central incisor and bicuspid 126. Class V amalgam 125. Larger than the real one C. A. Rocket bridge using central incisor as abutment B. When repairing a fracture of lower complete denture. Patient with class II division II. In regards to Electrical Vitalometer: A. Which statement is correct: A. D. C. A cusp fracture immediate to Class II inlay can be detected by. To test recently erupted teeth B. Greater reduction in smooth surface caries from in pit and fissures D. Reveal potential necrosis . The same size 123. B.121. You want to make a fixed bridge which of the following is suitable: A. Self curing will distort the denture B. Which of the following statement is correct? A. what would you do? A. Tooth decay declines by 90% to 95% B. Tooth decay declines by 45% to 55% C. Fluoridation is the adjustment of the fluoride content of a community water supply to optimum levels for caries prevention. Clinical /Proximal in some papers/ caries on radiographs are seen: A. Scrap the soften cementum and apply fluoride B. Fluoridation increases vulnerability to osteoporosis 122. Check response for an electrical stimulant C. E. Scrap the soften cementum and use GIC C. Smaller than the real one B. Cold curing will not be strong enough because of small area of attachment C. History Visually Radiograph Percussion Touching the tip of the cusp / Pressure on the cusp/ 124. the lateral incisor is missing. Recession of gingiva of several anterior teeth caused by exposure and softened cementum. Cantilever using central incisor C. 129. After 24 hours C. 130. C.128. Reconstructing the occlusal anatomy is based on: A. B. Necessity to restore normal anatomy 132. E. How do you prepare floor of pulp chamber in molars: A. D. B. B. C. D. Recklinghausen C. Functional factors B. Swab and dry with cotton wool and excavate Use round bur to flatten the floor Under cut walls Use flat end fissure bur to make it levelled When do you finish campsite resin restorations: A. Cemented pin 131. When preparing class III for composite restoration. Always should be performed to minimise marginal leakage Should not be performed because it might damage the adjacent tooth When extra retention is required Only in situations where cavity is shallow to avoid pulp irritation Death of the pulp Complete calcification of pulp chamber Hyperaemia Pulp stone All of the above Which pin system has proven to be the most retentive A. Immediately after curing B. A week after placement 134. Where Café au lait spots are seen: A. Von Willebrand’s disease B. In which situation the translucency of a tooth is lost: A. 133. Neurofibroma . Depth of restoration on a tooth C. which situation acid itching should be placed: A. Friction peak pin C. C. Self tapping threaded pin B. D. 135. What is the most common fracture of Class II amalgam restorations: A. C. D. Rotation movement B. Drugs contraindicated with Monoaminoxidas MAO A. 141. D. What is the advantage of composite over silicate resin: A. Blow to mandible causing fracture in molar’s right side region. B. you expect a second fracture of: A. Lingual movement C.5 to 1% 1. Isthmus because of insufficient depth B. Barbiturates Pethidine Local Anaesthesia with felypressin Narcotic analgetics Salicylic acid 138. A. Less shrinkage Less surface erosion Less water absorption All of the above The setting expansion of casting investment is approximately A.1 to 1. C. Von Willebrand disease is. Marginal ridge site 140. C.1 to 0. D. Haemophilic disease Bacterial Endocarditis Congenital cardiac disease Rheumatic fever What technique is used in the extraction of permanent 1st molars: A.6% .1% 0. Sub-condylar of left side C.5% 0. 136. Sub condylar of right side B. Internal fracture C. E. 0 to 0. B. B. C. Fracture of symphysis 139. B. D. Buccal movement 137. D. B. limited excision surgery because of the cosmetic considerations. B. Treatment of all of Giant Cell lesion either salivary or multiple is.142. The un-polymerized monomer in Self-cured resin is approximately:** A. D. 148. C. 0. Oil or water on impression for treatment casts causes: A. Excision and removal of adjacent teeth E. 147. 143. C.5% 5% 10% A volume shrinkage of methyl meta cyrelate monomer when is polymerized:** A. B. C. if it is large lesion. E. 144. Irradiation D.40% 3% 12% 15% 18% 21% Treatment of fibrous dysplasia consists of. A. 0.5% 1. C. D. Resection B. E. C. C. 145. The contraction of gold alloys on solidifying is approximately: A. B. Complete excision if it affects small area. A. D. Marsupialization In velation and packing ap?? Cold well?? Surgical curettage None of the above An increase of the quality No alteration A decrease of the quality Bubbles on the cast None of the above What is Path of Insertion .5% 2. D.5% 2. B. None of the above 146. To obtain a desired projection of occlusal loads. B. reaction. C. The appliances movement from the rest position to the last contacts of its rigid parts with the supporting teeth B. D. D. the floor of the occlusal rest should. F. a process that within limit: A. Areas to be shaped to properly loc?? Rigid parts of framework C. The transfer of stress by Tensile Action employs T. The movement of the appliance from the points of initial contacts to path of final rest position 150. Areas to be revealed as blocked out to properly loca?? Rigid parts of a frame work B. B. A. What is Path of Removal: A. Areas used for guideline planes D. C. When correction preparation for re contouring of occlusal surface is to be applied. Fails to promote bone growth Promote bone growth and maintenance Fails to promote maintenance None of the above 153. C. Areas used for retention E.A. E. B. Grinding only of the adjusted surface: A. Should not be felt flat Require a flat crown Require no contact with adjacent teeth Should be felt flat None of the above 151. Be convex Slope from the marginal ridge towards Contact?? of abutment Slope from Contact?? of abutment towards the marginal ridge Be concave Does not slope from the marginal ridge towards Contact?? of abutment None of the above 152. Areas used for support . The movement of the appliance from the points of rest position until it is not in contact with teeth 149. The movement of the appliance from the points of initial contacts to path of final rest position B. Which of the following arrears CAN NOT be determined by survey analysis of partially edentulous cast? A. E. D. B. Cool down the slab B. Prolong GIC’s setting time can be achieved by. Type I . Aids in balancing occlusion Assure predictable clasp retention Form right angle with the occlusal plane Eliminate the necessity for precision attachment Eliminate the necessity for a posterior clasp Rough surface of porcelain /Porosity/ is a result of: A. “The combined pericemental area of the abutment teeth should be equal to or greater in pericemental area than tooth or teeth to be replaced” 160. D. Increase the amount of distilled water 158. In regards to Gold casting alloys which one is available for bridge A. Rapid heating 157. Overjet C. The maxillary canine is missing. Ante in 1926 stated that. Lack of compression B. Overbite 161. Type II C. The best way for making Cantilever bridge: A. A. Improper metal framework B. Ante’s Law: Dr. Both premolars B. Depth of rest seats 154. Aesthetic B. The most common failure in constructing porcelain to metal is: A. Why would you decide to replace the anterior missing teeth for partial denture using bridge: A.F. Hard alloy “Type III” B. 155. A. E. Incisors and premolars 159. In partial dentures the guidelines “Guiding Planes” serve to: A. C. Sudden high temperature 156. Central buccolingually 170. The first thing to check when patient comes complaining of pain under denture is: A. is a landmark to determine the posterior bonds of upper denture: A. Distally B. Post dam B. Occlusion B. Are more resilient than plastic acrylic B. The position of cusps of maxillary first premolar during setting of teeth and on occlusal view is positioned:** A. Flanges 165. Tissue conditioning material : (Silicon lining material) A. Modulus of elasticity is defined as: A. The most common cause of RCT “Root Canal Treatment” failure is: A. Mesially C. The stress at the proportional limit B. Can minimise any bacterial colonies 168. Lost of teeth substance as a result of chemical exposure 166. The stress-strain ratio within the proportional limit 167. Using fluoride in the root surface caries is to protect. Over filled canals 169. Attrition is. Cuticle 163. Fovea Palatini. Dentine and cementum C. A. The middle of the root B. Soft tissues changes 164. Enamel B. Process of normal wear of teeth B.162. A. Lateral canals are usually found: A. The canal not filled completely (Short obturation) B. Fist third of the root close to the crown . E. Ameloblastoma B. the most likely diagnosis is. Squamous cell carcinoma C. The cause of development of lateral canals is: A. The apical third 171. Internal resorption C. D. A. Conditioner B. Transillumination is used to :** A.C. Osteosarcoma 174. Pumice and water B. Swelling after RCT is mainly caused by “Being asked as What is the most frequent cause of pain which occurs several days after obturation” too: A. or the presence of bacteria in the periapical region. Drainage through canals B. 173. The first step in the treatment of erosion is: A. Under filling the root canal system C. Replantation of avulsed tooth 2 ½ hours after incident. How do you treat dentine before applying GIC A. The emergency treatment for painless necrotic pulp is: A. External resorption B. C. To find intrinsic tooth colouration To detect caries Pulp-stones Hemorrhagic pulp Calculus What is the common malignant lesion that occurs in the oral cavity: A. Spray with Na-bicarbonate . Over filled root canal 177. Pulp stones 175. Entrapped Bacteria. B. None 176. Pumice & water 178. Cracks in Hertwig’s epithelial root sheath 172. B. 75mm B. Polyvinyl impression material are. Not to survey when making the crown . Should not be in touch at all 182. The undercut for Cobalt Chrome’s retentive arm clasp is. To remove the pulp tissue from narrow canal. A. What statement is false: A. Tilt the cast 187. The contact between artificial and natural teeth in partial dentures: A. Slight touch in the balancing side B. A. Barbed broach Small K-Type file Smooth broach Reamer Wax patterns ARE NOT to be left on the bench for long time because of. To increase the adoptability of occlusal rests 181. Lost of elasticity 185. GIC 179. Kine-matic face bow is used for recording (to locate) A. 0. The most stable B. To minimise the retention of plaque C. 184. To smooth the rough surface B.50mm C. Why do you polish the teeth before seating of partial dentures: A. Hinge movement (position) axis 180.25mm 186.C. The most resistant to heat 183. A. When surveying: A. C. D. Distortion B. 0. 0. you can use: A. B. To observe tooth from different angle 191. Painful 196. Alloy with high modulus of elasticity B. D. Retentive Clasps: A. but Co-Chrome has high modulus of elasticity A. A. What is main reason of ordering another Periapical radiograph of the same tooth: A. At the apex B. As far as you can obturate C. Clasp arm is gingivally located 195. 189. Use teeth with narrow Buccal-Lingual dimension B. Posterior teeth 190. 0. As close as possible to the gingival margins 193. A. Internal resorption of RC usually A. Gold clasp is more elastic than Cobalt Chrome. B. When doing pulpotomy with Formcresol. you will find: . Use mucco-compressive impression 194. C. B. The first statement is false the second is true Both are true The first is true the second is false Both are false Overdentures are best used for. To minimize the load on free end saddle partial denture: A. To disclose the other roots B. Below the survey line. Canines and premolars B. Above survey line C. Asymptomatic B.5 t0 1. Retentive part of clasp position is. A.5 mm before the apex 192. The ideal length of RCT is.188. Mechanical 203. Applying hypertonic Fluid on the dentine the transmission of fluid through tubules will be: A. Transmission of fluid in dentinal tubules is by: A. Lateral Condensation B. The technique of placing Gutta-Percha cones against the root canal walls providing space for additional Gutta Percha is termed: A. One major Gutta Percha point C. which part of flame we will use . Laterally above condensed 200. In melting gold. Corticosteroid 198. Angioneurotic oedema. One hypothesis of pain modulation is based upon the inhibitory-excitatory interaction of afferent fibre synapses. 204. the two most common micro-organisms are: A. Streptococcus and Staphylococcus 199. From outside to inside 202. Mummification 197. Hydrodynamic pressure (Osmotic) B. Antibiotics B. Puffiness around the eyes. negative thermal tests. There is no caries. Necrosis B. A. Epithelial rests of Malaise 201. From inside to outside B. Ledermix used in RCT to relieve pain because of. what epithelial cells you can find: A. In infected root canal. Caused by several deep restorations in the anterior teeth C. A. In periodontal membrane. negative percussion and negative response to palpation 205. Gate theory about pain control is: A.A. oedema of the upper lip with redness and dryness B. Contains micro filled B. A. the tissue may stick to the electrode because of . A.** A. The current intensity is too low 211. Weak edge strength and lack of surface details B. A. The lingual flanges should be concave 207. Subjected /undergone/ to cold treatment during processing (annealed) 214. The occlusal plane should be above the tongue C. Hybrid composite resin is used in posterior teeth because it: A. Back pressure porosity B. Quenched C. A. Where do you use the floss as a guide to the rubber dam: A. If the investment is burnout rapidly. Dimensional inaccuracy 209. The current intensity is too high B. Better colour matching 212. Cracking of the investment 208. The occlusal plane should be below the tongue B. Through the contacts. Oxidizing zone 206. Reduced zone B. Marble B. Proprioceptors 210. Tapering B. Long path of insertion 213. Overdenture advantage is. what will happen: A. Wrought metal is to be. The best way of getting good retention in full veneer crown is by. What is the DISADVANTAGE of gypsum dies:** A.A. To increase the stability of the lower denture. . In electro surgery. Mastication Tooth shape Tooth inclination and crowding Salivary flow . B. Infra bony pocket B. D. B. A. Vertical bone resorption C. In periodontitis. Periodontal membrane C. 219. Acute periodontal abscess Chronic periodontal abscess Apical abscess Chronic alveolar abscess 216. C. A. D.215. C. Alveolar bone B. D. Hyperplasia of the gum 221. E. Horizontal bone resorption B. Angular bone loss 217. What DOES NOT prevent the calculus formation “build up”: A. 1-2 mm 0-3 mm 2-3 mm 0-5 mm Fibroblast Epithelial cells Erythrocytes Vest cells of malaise Inflammatory plasma cells and lymphocytes The term false pocket stands for. “Main feature of suprabony pocket” A. Periodontitis occurs in. The normal range of gingival depth “Epithelial attachment” in healthy mouth is: A. D. Loss of periodontal attachment C. C. In young children what is the commonest finding after dental complaint: A. The commonest elements which are found in periodontal membrane are:** A. B. C. the most common finding is. 220. B. Alveolar bone and gingiva 218. Surface grinding followed by fluoride application Surface grinding followed by GIC restorations Class V cavity preparation for a GIC preparation Cavity preparation for amalgam preparation Application of fluoride without surface preparation Which of the following is not useful for apical infection: A. D. D. what your first decision would be: A. 226.E. E. Electrical pulp testing is least useful in /or does not detect in some papers/ . D. Which of the following would be your preferred procedure: A. Chlorhexidine H2O2 EDTA Ethyl alcohol Eugenol 224. D. E. Patient presents to you with remarkable resorption of gingivae around the remaining teeth. B. what is the first thing to consider when you get a patient with intruded 11 and 12: A. E. Oral flora 222. A child with fracture of tooth at the apical third of the root. B. Wait and recall after one month and observe for any necrotic or radiolucency Root canal treatment Extraction Apiectomy 225. 227. 223. C. C. C. In the pulpchamber under mesiolingual cusp . some areas of cementum appears to be soft. The oral hygiene is not good. C. B. C. D. B. mainly around the lower bicuspid and anterior teeth. Replace intruded teeth in position Advice patient about consequences Leave it and observe X-ray Traumatised teeth Just erupted teeth Multi-rooted teeth Capped teeth Necrotic pulp The palatal pulp horn of maxillary molars is located: A. A. B. The most characteristic allergic reaction to drugs is. General anaesthesia A. Toxicity as a result of anaesthetic solution can be seen more when: A. D. none of the above All of the above I and II II and III I. In the pulpchamber opposite the mesio distal fissure of the buccal cusp C. E. Excitement III. Acetyl salicylic acid . C. B. Skin rush with swollen of lips and eyes 229. Sedation II. Under the disto lingual cusp 228. Injection in supine position Injection into vascular area Injection without vasoconstrictors Intravenous injection 233. IV and V 231. Analgesia IV. A. Diabetics B. Pethidine IV. Antibiotic prophylaxis should be used for patient with. When taking Mono Amino Oxidase Inhibitors (MAOI). Opioid analgesics reduce pain by the release of which naturally appearing product: A. Which is not an effect of : I. Barbiturate II. Serotonin B. Histamine C. D. B. Rheumatic fever 230. Hypnosis V. Enkephalins 232. C. A.B. which are is contra indicated: I. Local anaesthetic III. 236. B. D. what is the possible diagnosis: A. Pressure on the genioglossi Mylohyoid muscles 238. Ranula B. Which of the following may be caused by newly placed restoration which interferes with the occlusion A. Apical periodontitis 235. A. Nerve to masseter C. C. the most common cause is. which of the following would expect to find: A. Patient with lower denture and complaining of paresthesia of the lower lip. Pressure on mental foramen B. Removal of enough bone Preoperative assessment The flap design The use of general anaesthetic The most important indication of malignant lesions is: A. C. Uni lateral swelling in the floor of the mouth occurs frequently with meal. III and IV “check Q137 too” 234. The nerve supplies TMJ is. C. Apical abscess B.A. In cleidocranial dysplasia. All of the above None of the above I. D. Pulpal necrosis C. II and III II. Sub-mandibular sialolith . Auricula Temporal Nerve B. Multiple un-erupted teeth and pseudo anodontia 240. B. Early lose of primary teeth B. B. The most important factor in surgical removal of impacted teeth is. Pain Paresthesia Teeth movement Tooth resorption 237. Facial nerve 239. A. A. D. Pemphigus Herpes simplex Aphthous ulcer ANUG Erythema migrans Erythema multiforme 242. Mucocele 241. 43 What is the percentage of leukoplakia that turn into cancer:** A. D. D. 22 35. Tetracycline B. C.C. 247. C. 245. E. 10% C. 45 15. Which two of the following conditions present as complete vesicles A. B. Keratosis follicularis C. An oral prodromal signs of Rubella are:** A. Cyst D. Keratotic lesion surrounded by cold web like lines /Wickham’s Striae/ appears as lace-like network on the buccal mucosa. White sponge nevus 243. Lichen Planus B. Fordyce’s spots Koplik spots Geographic tongue None of the above Which of the following conditions is not classified as a white lesion: . 25 33. D. 5%-6% B. Systemic penicillin C. F. B. you diagnosis is** A. B. How would you treat Denture Stomatitis A. 12. What are the commonest congenitally missing teeth: A. C. 25% 246. Nystatin + 244. The absence of lamina dura in radiograph is a feature of all of these except for:** A. 248. D. 254. Pain on percussion Pain when eating hot food Pain when eating cold food The thickness of periodontal ligament on X-Ray 252. Regional lymph node B. Give glucose Administer O2 Administer adrenaline Inject insulin Excision Excision and extraction of teeth Radiation Surgery and radiation In which direction you would extract a deciduous upper molar: . Cellulitis 251. B. Deficiency of . E. D. B. C. Fever C. Low vertical dimension 249. what would you do: A. Diabetic patient with moist skin. C. B. D. C. Fordyce’s granules Smoker’s keratosis Leukoplakia Lichen planus Angular cheilitis in edentulous patient with complete denture is a result of: A. 253. C. 250. C. vitamin B. How would you diagnose a periapical abscess: A. moist mouth and weak pulse. B. B. Paget’s disease Hyperparathyroidism Fibrous dysplasia Osteogenesis imperfecta Hyperthyroidism Which is usually found when a systemic infection is present: A. D.A.. How would you treat Epidermoid Carcinoma: A. D. E. D. Hydrolysis of the filler phase of the composite Hydrolysis of the resin phase of the composite Bacterial acid formation dissolving the enamel Salivary pellicle growth at the interface Setting contraction of the composite resin The optimum cavosurface angle for occlusal amalgam surface is: A. Lingually 255. 258. C. C. E. C. C. Rotation B. B. Impression without elastomer in custom tray has been taken for crown preparation. B. B.A. Polyether Thiokol or meraptan rubber Condensation silicone Vinyl polysiloxane 256. Buccally C. D. E. 45-60° 70-85° 45-80° 95-110° 130-150° 259. E. A major difference between light cured and chemical cured composite is that during setting or in function the light cures material tends to: A. D. Which impression material is preferred? A. D. B. D. Seal the margins better and completely Exhibit less wear on time Undergo greater colour change Shrink rapidly Posses greater fracture toughness . sequentially triturated Several small mixes with varying mercury/alloy ratios A basic mix to which additional mercury is added as needed 257. what type of amalgam mix would you prefer: A. Micro-leakage at the attached enamel-composite resin interface is most likely to be due: A. C. A large amalgam core is to be condensed around several pins in a vital molar tooth. B. it will be two days before impression gets to the laboratory for construction of the crown. A large mix to ensure homogeneity A large with extra mercury to give easier manipulative qualities Several small mixes. B. C.260. The bur should be tilted lingually when preparing the occlusal surface of class II cavity on a mandibular first premolar in order to: I. I and II I and III II and IV III and IV IV only 263. 2mm reduction while forming a flattened surface 2mm reduction while following the original contour of the cusps 4mm reduction while forming a flattened surface 4mm reduction while following the original contour of the surface 262. E. B. D. C. D. the more number of cutting blades and low speed will result in: A. C. 261. an incipient carious lesion limited to the end of the proximal surface of posterior tooth appears as: A. In regards to carbide burs. B. If the sealant of bonding agent is not placed on part of enamel that has been etched by an acid solution. Less efficient cutting and a smoother surface Less efficient cutting and a rougher surface More efficient cutting and a smoother surface More efficient cutting and a rougher surface For an onlay preparation during the restoration of a tooth. C. which one of the . Maintain dentinal support of the lingual cusp A. you would expect: A. D. E. D. Prevents encroachment on the lingual pulp horn IV. B. Prevents encroachment on the buccal pulp horn III. D. Arrest of enamel carries by organic sulphides The enamel is to return to normal within 7 days Continued enamel declassification in the etched area Slight attrition of the opposing tooth When restoring weakened cusps with dental amalgam you should consider: A. C. 265. Remove unsupported enamel II. B. Radiopaque area Triangle with apex towards the tooth surface Larger in radiographs than actual lesion All of the above None of the above 264. In radiographs. one of them is tin 267. tin. Amalgam is an alloy of two or more metals. copper and zinc B. Refer him to physician Anaesthetise all of the maxillary left anterior teeth to provide instant relief Give him an ice pack to be placed on the area to control the swelling Take radiograph and test vitality of his teeth Write prescription for antibiotics and delay treatment until swelling is reduced The prognosis of tooth with apical resorption is : A. At which angle to the external surface of proximal cavity walls in a class II preparation for amalgam should be finished A. E. B. The term TUGBEN?? is related to : “When used in connection with a master Gutta Percha cone in endodontics” . one of them is mercury D. Poor Good if apex can be sealed Dependant upon periapical surgery Contingent upon systemic antibiotic therapy combined with treatment of the canal 270. D. 266. D.following is the MOST EFFECTIVE means for verifying adequate occlusal clearance A. fluctuant and pointed on the labial plate under his lips on the left side. the corner of his nose and a region under his left eye. The swollen area is soft. B. Amalgam is a metallic substance in powder or tablet from that is mixed with mercury E. C. Amalgam is a metallic powder composed of silver. Teenager has swelling involving his upper lip. B. B. C. D. 269. What is the first thing you would do after taking history and temperature: A. C. Amalgam is an alloy of two more metals that have been dissolved in each other in the molten state. Wax bite chew in Proper depth cuts Visual inspection Articulating paper Choose statement that correctly defines the term AMALGAM: A. Amalgam is an alloy of two or more metals. C. C. D. An acute angle An obtuse angle A right angle An angle of 45° 268. His body temperature is 39°. Palatal reduction may be of minimal thickness Overall conservative for tooth structure Ability to watch the appearance of adjacent natural teeth Less laboratory time . The main factor controlling a decision to increase the occlusal height of teeth for extensive oral reconstruction is whether. B. E. D. B. Tensile strength of the gutta percha Consistency of gutta percha Size of the cone Fit of the cone in the apical 1 or 2 mm Length of the cone One canal with one foreman One or two canals with one foreman Two canals with one foreman Two canals with two foremen The most common cause of porosity in porcelain jacket crown is. the aesthetic appearance of the patient will improve sufficiently to warrant the planned reconstruction 275. Moisture contamination Excessive firing temperature Failure to anneal the platinum matrix Excessive condensation of the porcelain Inadequate condensation of the porcelain 274. the inter occlusal distance will be physiologically acceptable after treatment B. B. C. 272. B. B. D. C. A. D. there will be sufficient tooth bulk in the abutment teeth for proper retention of the crowns C.A. C. at least two third of the original alveolar process will remain for adequate periodontal support D. E. An advantage of metal-ceramic crowns. 273. C. Should extend to the level of the apex to minimize irritation Should extend slightly through the apex to ensure a complete seal Should extend to the dento cemental junction for healing The extension of the filling is not critical Mesiobuccal root of maxillary first molars MOST COMMONLY have: A. A. D. In root canal therapy it is generally accepted that the ideal root filling. A. compared wit full ceramic crowns for restoring anterior teeth is. A. C. 271. D. The minimal labial tooth reduction for satisfactory aesthetics with porcelain fused to metal crown is. B. Lighten the colour of the teeth by the opacity of the cement Darken the colour of the abutment by the presence of metal on the lingual Have no detrimental colour effect Darken the abutment teeth by incisal metal coverage 277. C.276. which of the followings terms is NOT CORRECT: A. Hand burnishing Mechanical burnishing Using finishing burs and points to remove the enamel margins on the tooth Making a new impression and remaking the crown Relieving the inside of the occlusal surface of the casting to allow for further seating 281. B. Rigid components anterior to the premolar teeth B. A crown casting with a chamfer margin fits the die.5 mm 2. The saddle is the area of the edentulous ridge over which the pontic will lie and comes in contact with pontic D. 279. but in the mouth the casting is open approximately 0. the minor connector refers to:** A. A satisfactory fit and accurate physiological close of the gingival area of the crown can BEST be achieved by: A. A pontic is an artificial tooth as part of a bridge 280. A connector connects a pontic to a retainer or two retainers to each other C. Flexible components. in contrast to rigid major connectors . The gingival portion of natural tooth differs in colour from the incisal portion because the : A. D. D. In cementing Maryland or Roche bridges. E.3mm.5mm One third of the dentine thickness 278. the effect is generally to. A retainer could be a crown to which a bridge is attached to B. 1mm The full thickness of enamel 1. A. B. B. C. E. When describing a removable partial denture. C. Lighting angle is different Gingival and incisal portions have different fluorescent qualities Gingival area has a dentine background Incident light is different In bridge work. D. C. A. D. C. the principle of an indirect retainer is that: A. C. B. It is used when the space between raised floor. D. C. D. B. C.C. the retentive clasps tips should: A. Apply retentive force into the body of the teeth Exert no force Be invisible Resist torque through the long axis of the teeth Why do you construct a lower removable partial denture with lingual bar: A. 287. Ductility was too low Hardness was too great Ultimate tensile strength was too low Tension temperature was too high Elastic limit was exceeded 285. Which of the following is true regarding preparation of custom tray for elastomeric impression: A. E. D. B. Smaller connectors which connect denture components to the major connector D. The means by which one part of a partial denture framework opposes the action of the retainer in faction is:** A. Adhesive is preferred over perforation Perforation provides adequate retention Adhesive is applied immediately before procedure Perforations are not made in the area over the prepared tooth 286. D. Distortion or change in shape of a cast partial denture clasp during its clinical use probably indicates that the: A. B. B. C. 283. D. Plaque accumulation is less than lingual plate . When a removable partial denture is terminally seated . The components of the denture base which provides reciprocation 282. mouth and gingival margin is minimal B. Tripoding Reciprocation Stress breaking Indirect retention In removable partial denture. Stabilise against lateral movement Prevent settling of major connectors Restrict tissue movement at the distal extension base of the partial denture Minimise movement of the base away from the supporting tissue 284. It is a soft film composed mainly of dextran and can not be rinsed off the teeth E. The retained roots are covered by the denture thus protecting them from caries and periodontal diseases 291. It is a soft film composed mainly of none calcified bacteria and can not be rinsed off the teeth D. It is a soft film composed mainly of food debris and can not be rinsed off teeth B. It is a soft film composed mainly of dextran and can be rinsed off teeth. Located on either sides of the midline close to the junction of the hard and soft palate D. Von Willebrand’s syndrome 293. Retention and stability are generally better than with conventional complete denture C. Amalgam B. Alveolar bone resorption is reduced D. It is a soft film composed mainly of food debris and can be rinsed off teeth C. Composite resin 290. B. Trauma to extraction site Increased the potential of infection Impossibility for anterior try in Excessive resorption of residual ridge Brown skin pigmentation does not occur in: A. 294. C. Which statement BEST prescribe plaque: A. Which of following restoration material its strength is not effected by pins: A. 292. Which of the following is a major disadvantage to immediate complete denture therapy: A. Foramina covering the lesser palatine nerves and vessels B. Morphologically related to the formation of the premaxilla C.C. Greater occlusal loads can be applied by the patient B. Hyperparathyroidism B. Should be make thicker when short 288. The gingivae of child is diagnosed on the basis of all of these except of: . Closely related to the rugae of the palate 289. D. The Fovea Palatinae are: A. Which one of following statement about Overdenture is not correct: A. B. D. Alginate> Polysulphide> Silicone> Zinc Oxide Eugenol Silicone> Alginate> Polysulphide> Zinc Oxide Eugenol Alginate> Polysulphide> Zinc Oxide Eugenol>Silicone Alginate> Silicone> Polysulfide> Zinc Oxide Eugenol Alginate> Zinc Oxide Eugenol> Silicone> Polysulphide 300. the powder is referred to as. Rank the following impressions materials according to their flexibility A. C. B.** A. C. 295. D. In regards to the glass of quartz particles of filling restorative resin. D. A. C. D. E. The maximum stress under tension that can be induced without failure The maximum elongation under tension that can be measured before failure The minimum stress required to induce permanent deformation of a structure Minimum stress in structure Maximum strain that can be measured. A higher coefficient of thermal expansion and a higher crashing strength A higher coefficient of thermal expansion and a lower crashing strength A lower coefficient of thermal expansion and a higher crashing strength A lower coefficient of thermal expansion and a lower crashing strength Mercury is dangerous when it turns into vapour form because of. 298. C. E. The remnants of Ameloblast contribute to the primary enamel cuticle the last secretion of the odontoblast is cementum X The last secretion of the ameloblast is the acquired of enamel cuticle The remnants of odontoblast form the primary enamel cuticle 296. D. C. B. B. Contour of gingival papilla Sulcus depth Contour of Nasmyth membrane Tight filling of gingival collar Which one of the following statement is correct. 299. 297. A. Denture resin are usually available as powder and liquid that are mixed to form a plastic dough. D. the microfill resins tend to have. B. A. Initiator . A.A. C. B. It is accumulative and cause liver poison It is accumulative and cause kidney poison It induces neoplasia in the liver It is accumulative and cause brain poison It induces neoplasia in the brain The elastic limit may be defined as the **. E. B. C. D. E. Polymer Inhibitor Monomer Dimer 301. Which one of the following is the major disadvantage of stone dies used for crown fabrication, A. B. C. D. 302. They lack accurate reproduction of surface details Their overall dimensions are slightly smaller than the original impression The strength of the stone The hazard of aspiration of toxic materials during trimming of the dies. Glass Ionomer Cement sets because of,** A. B. C. D. E. 303. Acid-Base reaction Addition polymerisation reaction Growth of glass crystals Slip plane locking Solvent evaporation The articular surface of the normal temporomandibular joint are lined with, A. B. C. D. A specially adapted, highly fibrous tissue Hyaline cartilage Chondroitin-6-phosphate Highly vesiculated tissues 304. When all other removable partial denture consideration remains unchanged; clasps constructed of which material can be engage the deepest under cut: A. B. C. D. Chrome cobalt casts Nickel chrome casts Wrought stainless steel Wrought gold 305. Which one of the following types of pain is most likely to be associated with cranio mandibular disorders: A. B. C. D. 306. Exacerbated pain by hot or cold food Keeps patient awake at night Associated with muscle tenderness Associated with trigger spots related to the trigeminal nerve The incisal guidance on the articulator is the:** A. Mechanical equivalent of horizontal and vertical overlap of upper and lower incisors B. Mechanical equivalent at the compensating curve C. Same as condylar guidance D. Estimated by the equation: Incisal guidance = 1/8 of condylar guidance 307. When immature permanent molars that have been treated with Ledermix pulp capping, the most probable pathology is, A. Chronic inflammation of the pulp B. Necrosis of the pulp 308. Child with rampant caries taking medicine with high quantity of sugar; the best way to help preventing caries is, A. B. C. D. Change sugar to sorbitol sweetener Report the patient is having expectorant Give him the syrup during sleep time Give him inverted sugar 309. How many ppm “ Part Per Million” of fluoride are present in water supply in case of temperate climate:** A. B. C. D. 310. The difference between deciduous and permanent teeth are: A. B. C. D. 311. Deciduous teeth have a higher pulp horns and larger pulp chambers Deciduous teeth have flatter contact areas Deciduous teeth have thinner enamel surface All of the above The most resistant filling materials to fill class IV cavities are: A. B. C. D. 312. 1 ppm 2 ppm 8 ppm 1.2 ppm Resins with silicone dioxide (SiO2) Resins with glass or quartz Silico-phosphate Silicates With dentin bonding agent, you apply: A. First acid etching to dentine and then bonding agent B. Bonding agent directly to dentine C. Chelating agent (EDTA) and bonding agent 313. What is the best way to apply aspiration before injection: A. B. C. D. 314. Short, sharp pressure backwards Pressure for 2 to 3 seconds Long pressure Turning the needle 90° between two aspirations The method you will use to fill root canal of maxillary lateral incisor is: A. One major Gutta Percha cone B. Laterally condensed C. Laterally above condensed 315. What controls the occlusion:**?? Check Boucher C.D. A. B. C. D. E. F. 316. Teeth Receptors in periodontal membrane Proprioceptors Neuromuscular receptors TMJ All of the above How would you extract 35? A. Rotation B. Lingually C. Labially 317. Why the method of extracting lower 8’s by directing the extraction lingually is used:** A. Because of the roots direction B. Thinner bone C. Lingual deviation 318. What the maximum dose of 2% lignocaine without vasoconstrictors:** A. B. C. D. 319. 5 ml 10 ml 50 ml 100 ml Where do Maryland bridges lose retention often: A. Resin-metal B. Resin enamel C. Resin layer 320. What is the function of gypsum-binder in the investment:** Zinc Oxide and eugenol ZOE . B. E. D. Pulp capping B. Where is the retentive position on tooth according to the survey line: A. What factor do you consider the most important when storing the occlusal part of a tooth: A. Produce acid Produce caries Produce chelation Not necessarily produce acid Treatment of gangrenous tooth: A. Pulpotomy 328. Buccal pockets Lingual pockets Mesial pockets Distal pockets Sinuous 325.A. In regards to distal free end saddle. C. Which material is not compatible with composite resin: A. Which periodontal pockets are evident on periapical x rays: A. Bad positioning of the occlusal rests C. Strength and rigidity 321. Below the height of contour B. Setting and hydroscopic B. Root canal therapy C. D. Function 326. what is TRUE: A. Improper survey B. B. C. Occlusal anatomy B. 327. What are the most common errors when constructing partial denture: A. Incorrect design 324. All dental plaque:** A. Will require relining more often than a denture supported with teeth 323. Next to gingival margins 322. The finger pressure is enough for mobility diagnosis A communicable disease X ray after intra alveolar surgery is sufficient for diagnosis healing Systemic diseases have no effects on it . D. B. D. D. B. C. Carboxylate D.B. 333. Diazepam Carbamazepine (Tegretol) Ergotamine Phenytoin 331. Ca(OH)2 C. Mesio angular horizontal Too big vertical angulation Too small vertical angulation High angulation Which of the following is false in regards to Cleft-Palate? A. Which Nerve is anesthetised in anterior border of ramus and 1 cm above occlusal plane of lower posterior teeth: A. Lingual nerve B. C. C. May be submucous More common in males than females Predispose to speech defects. B. Bone resorption Necrosis of the pulp Hypercementosis Triangulation All of the above Which drug is specific for Trigeminal Neuralgia: A. B. orthodontics problem and hearing loss Patients are more likely to have cardiovascular defect than the general population. 334. 330. Which of the following statement is correct for a periodontal disease:** A. C. Long buccal nerve 332. D. C. D. the mesio buccal root of upper first molars is elongated which is the result of: A. E. Zinc phosphate cement 329. In an X ray. Tooth under occlusal trauma shows: A. B. C. A. D. A. Contact area 340. 336. Slightly compress soft tissues B. E. B. 24 hours after application Immediately after application 3 to 4 days 3 to 4 weeks Not at all 341. Short Long Narrow Wide Which is NOT characteristics of canal filing materials “obturation material” A. Class V composite resin restorations can be polished. A.E. Zinc oxide eugenol cement and amalgam . 339. The major cause of jacket crown breakage is. ZoE paste will accelerate healing 335. Labially displaced anterior tooth is restored with a gold core porcelain jacket crown so that it is in line with the arch. the crown will appears: A. on x rays you find dent in dent. Caries which is close to the pulp chamber. Where is the biggest thickness B. B. C. Just in contact with soft tissues 337. B. D. Tacky adhesive to walls Radio opaque Not irritating Quick in setting The best location of pin in class II inlay is. C. D. Be clear of soft tissues C. 338. Inclusion of platinum foil Use of weak cementum Voids of porcelain Porcelain is thinner than 1mm Pontic replaces upper first molars in a bridge should be: A. C. B. the right treatment is: A. Mesial and distal angle C. D. Acids 343. D. D. Pulpectomy C. D. 344. D. Chelation B. The main advantage of amalgam with high content of Cu is: A. Dental plaque produces: A. C. B. Dental caries C. 345. C. B. B. Pulpotomy D. Upper premolar with MO cavity.What is complicated by” A. what is important about the application of the matrix band: “the question has shown too as …. C. C. D.5mm 347. E.5mm 2mm 2. 1mm 1. Better marginal sealing Less corrosion Better tensile strength Higher and immediate compressive strength The major disadvantage of self-threaded pin is: A.B. I II IV III V How much space do you need to cap a weakened cusp with amalgam: A. The mesial concavity of the root surface Small lingual pulp High buccal pulp horn High lingual pulp horn Concavity of distal root surface . E. Calcium hydroxide on pulp and amalgam 342. 346. B. B. Friction locked Too expensive Not all sizes available May cause tooth cracking In which class of cavities do composite restorations show most durability: A. C. for aesthetic considerations 349.348. how do you establish reliable vertical dimension. Wax if the remaining teeth occlude 355. Pain B. The most common characteristic symptom of malignant tumours occurring in lower jaw is. Why Class IV gold can not be used in cavity as a filling material: A. A. Bleeding C. Agranulocytosis B. 350. Dryness of the mouth Dryness of the eyes Rheumatoid arthritis All of the above Long use of Tetracycline is characterised by:** A. Hard 18% B. Low temperature solidifying point B. C. The type of gold that used for dental bridges is. The corrosive properties 353. the most uncommon result is:** . Can not be polished “burnished” B. Higher flow in room temperature 356. Candida Albicans 351. minimise the leakage of restorations B. In regards to indirect compare to direct wax technique:** A. A. B. Paraesthesia 352. In regards to Partial dentures. Hard in room temperature C. D. If amalgam gets contaminated with moisture. Type IV 75% 354. Sjogren syndrome is characterised by: A. Etching techniques are used always to: A. A. A. Erythema migrans /Geographic tongue/ 359. Paget’s disease shows in the early stages in jaws: . Band of lymphocytes inflammation and hyper parakeratosis C. Immunofluorescence of liquefied layer 363. Smooth rete pegs B. Blister formation Post operative pain Secondary caries Lower compressive strength Loss of contacts Slight tilting Pocket formation TMJ problem All of the above Which is not a malignant lesion: A. D. 7 and 6 361. 8. Long buccal nerve 360. A. 357. Impetigo 362. patient complains of itching and vesicalis on the upper labium (Vermillion region) every year. D. Leukoplakia B. B. your diagnosis would be: A. Mycostatin 364. Recurrent ulceration aphthae C. Tetracycline lozenges C. Anaesthesia 1 mm above last lower molars will anesthetise: A. The effects of tooth removal in healthy individuals can show as. 358. What is the typical feature of Lichen planus:** A. E. 8. Amphotencin B. Denture stomatitis is treated with. B. 7 and 6 except the mesio buccal root of 6 B. Herpes simplex B. C.A. C. Lingual Nerve B. Posterior superior alveolar nerve supplies:** A. B. B. Diagnosis of oral candidiasis (candidosis) is BEST confirmed by: A. Intermittent pain 368. B. Radiographs Electric pulp test Biopsy Thermal The most prominent feature of acute apical periodontitis is: A. C. Cysts C. 367. C. C. D.A. Microscopic examination of smears Biopsy Blood count Serological exam 370. Tenderness of tooth to pressure B. D. B. Marsupialisation is a technique used in the treatment of: A. Tetracycline C. Streptomycin . Penicillin B. Abscesses 369. Extra oral swelling C. D. cysts and chronic periapical abscesses may mostly be differentiated by : A. Pericoronitis B. Cotton wool Ground glass Orange peel Beaten copped 365. Granulomas. Which antibiotic administered in childhood may result in tooth discolouration: A. D. The most serious complications which may occur from abscess of max canine is: A. Cellulitis Cavernous sinus thrombosis Lacrimal duct stenosis Damage to infra orbital nerves 366. C. 371. Thyrotoxicosis 374. A patient with long standing rheumatoid arthritis and a history of steroid therapy. At the mandibular symphysis 372. Amoxicillin 2 gram an hour before operation orally B. In the maxilla C. Patient will need haematological evaluation 373. oral examination shows numerous yellow grey lesions. C. Patient is more susceptible to infection B. An adult patient with a history of bacterial endocarditis requires prophylactic administration of antibiotic prior to removal of teeth. B. What is the MOST LIKELY diagnosis: A. D. she has painful cervical lymphadenitis and a temperature of 39°c. he presents for multiple extractions. D. C. Measles Erythema multiform Herpetic gingivostomatitis Stevens-Johnson syndrome The causative micro organism for Herpetic gingivostomatitis is: A. Congestive cardiac failure C. Tetracycline 250-500 mg orally 2 hours before treatment 375. indicate the pre-operative regimen:** A. Penicillin 250 mg orally six hours before operation C. until a week ago. Anxiety B. Near the angle of the mandible B. you should follow all of the following EXCEPT: . Ameloblastoma occurs MOST frequently: A. To reduce the side effects risk of local anaesthetic injections. The dentist should consult the patient’s physician because: A. A patient whose hands fell warm and moist is MOST likely to be suffering from:** A. B. Herpes simplex bacteria Herpes simplex virus Herpes zoster virus Borrelia vincentii 377. A 12 year old girl complains of sore mouth. 376. Patient may have a suppressed adrenal cortex C. C.A. Antibiotics should be used routinely to prevent infection arising from oral surgery in patients suffering from all the following EXCEPT: A. D. Nitrous Oxide (N2O) is not used alone as a general anaesthetic agent because of:** A. C. D. E. D. C. X-Ray Periodontal probe / Calibrated probe/ Periodontal marker Bitewing radiograph Sharp explorer Study cast The final material you use for endodontically treated deciduous molars is:** . B. Adverse affects on liver C. B. B. 12 times a minute 24 times a minute 50 times a minute 80 times a minute 381. D. The most potent viricidal properties: “another format of the same answer: Indicate which of the following has viricidal properties” A. F. B. D. E. Aspirate before injection Use the smallest effective volume Use the weakest efficient percentage strength Inject rapidly 378. 383. B. Sodium hypochlorite Chlorhexidine Glutaraldehyde Alcohol 70% Quaternary ammonium 379. Agranulocytosis Sever uncontrolled diabetes Aplastic anaemia Mumps Leukaemia At what rate is closed chest cardiac compression should be in an adult:** A. C. Difficulties in maintaining an adequate O2 concentration B. E. C. Poor analgesics affects 382. How can a periodontal pocket be recognised:** A. 380. 388. B. C.A. 389. E. Mast cells Polymorphonuclear leukocytes Eosinophils Epithelial cells The presence of sulphur granules is diagnostic of:** A. D. C. Actinomycosis Candidosis Viral infection Keratocyte Immediate aim of dry socket treatment is to:** A. C. D. Avoid Osteomyelitis B. C. Control pain 387. B. D. Which is the LEAST likely to cause Xerostomia: A. Sjogren’s syndrome 390. Heart disease . 384. D. Mumps C. D. Amalgam GIC Composite resin Wrought base metal crown Which type of cells does an abscess contain: A. Sjogren’s syndrome Emotional reaction Antidepressants drugs Submandibular sialolith Herpes simplex infection Oral lichenoid reaction Aphthous ulceration Pemphigus vulgaris Cicatricial pemphigoid Painful salivary gland are MOST likely to be indicate to:** A. A patient with an acetone odour would be suspected suffering from: A. 385. Intact vesicles are MOST likely to be seen in:** A. B. B. 386. C. B. Mucocele B. Chronic inflammatory periodontal disease originates in: A. Which is the most important local factor in the aetiology of periodontal disease: A. Loss of arch length B. D. C. Which of the following does state BEST the morphology of periodontal ligament fibres: A. Occlusal trauma Calculus Brushing habits Coarse food 393. D. B. Alveolar bone Whole face Mandible Maxilla 396. Diabetes 391. D. B. MOST common consequence arising from premature extraction of deciduous molar is: A. Liver damage C. E. Loss of facial contour . Which of the following is LEAST to cause toxicity from local anaesthetic injection: A. D. 395. Cervical cementum 392. C. Elastic Striated Non striated Levity Wavy 394. The marginal gingiva B. C. Loss of speech sound C.B. C. B. B. this would MOSTLY affects the growth of:** A. The crystal alveolar bone C. Injecting in supine position Injecting in vascular area Injecting without a vasoconstrictor Intravenous injections If a child’s teeth do not form. B. B. Which of the following figures is incorrect** A. Between canines C. The amount of fluoride required to reduce caries according to age and level of fluoride in drinking water. C. 401. D. 2. Distal to the first molar 398. B.2ml B. Open bite Retrusion of maxillary central incisors Reduced Overjet Increased overbite 399. 1 year old child requires no fluoride when the fluoride in drinking water is 0. 6 years old child requires 1mg of fluoride when drinking water containing 0.5% phosphoric acid Use NH4F instead of NaF Which of the following ahs the highest sucrose content: A. C. the greatest increase in the size of the mandible occurs: A. Several application has been suggested to increase the effectiveness of prophylactic application of topical fluoride which include all EXCEPT: A. 22ml 400. The major etiological factor responsible for Class II division 2 malocclusion . C.7PPM C. When injecting without vasoconstrictor. At the symphysis B.397. D.5mg 403.3PPM B. Ice cream Canned juice Cough syrups Breakfast cereal Sweet potato 402. D. 3 years old child requires no fluoride when the fluoride in drinking water is 0. After the age of 6 years. the maximum safe dose of 2% lignocaine solution for 70Kg adult is: A. E. Increase Fluoride ions in solution “increase concentration” Increase PH of fluoride Increase exposure time to topical fluoride Pre-treat enamel with 0. E. Which is present in Angel’s Class II division 2 malocclusion: A. The periapical area 408. Pulpotomy B. 2 3 4 5 All of the following are keratinised EXCEPT of: A. How many pulp horns are presented in a typical mandibular deciduous second molar: A. C. Mesial inclination of the 1st permanent molar B. D. Preschool child has an intruded upper incisor. C. Ankylotic primary second molar in the mandible is not always a good space maintainer because of: A.in Angel’s classification is:** A. be** A. Thumb sucking Growth discrepancy Tongue thrust habit Tooth to jaw size discrepancy Skeletal cause (discrepancy) 404. B. Where is the MOST probable place of bone resorption after a deciduous molar has a pulpal gangrene: A. Endodontic treatment C. It does not keep up with the rest of occlusion 405. 406. 409. B. What treatment would you do to the deciduous tooth: A. An upper deciduous molar has a caries exposure and on X ray the corresponding 2nd permanent premolar is absent. D. Crevicular epithelium . D. Interradicular septum B. Pulp capping 407. E. B. C. what would your treatment X-ray Put it back in place and splint Control bleeding and check after a month Make the patient comfortable without disturbing the tooth. Calculus Plaque Caries Restorative material How can you improve the adhesion of a fissure sealant: A. 410. In class II restoration. D.B. D. Saccharin C. 6 year old child who had a history of primary herpes simplex has got a recurrent infection. B. Remove plaque and debris in interproximal surfaces Polish Massage of the interdental papillae Aid and recognise subgingivally 413. C. Mannitol B. E. irritation is: A. Several months or longer 414. What is the likely cause: A. Which one of the following is a non-calorie sweetener: A. C. Xylitol 415. Few weeks B. D. all of the following considered to occur as probable causes of periodontal problems except: . Palatal epithelium Alveolar mucosa Free gingiva Attached gingiva The MOST cause of gingiva. Thickening of the periodontal membrane 417. The advantage of using dental floss over rubber point interdentally: A. 411. Herpes labialis 416. C. Acid etching technique 412. What will be the periodontal response: A. After prophylactic treatment. B. you decide to change the flora to a nonacidogenic by changing the diet. How long does it take to achieve this change: A. A newly placed restoration interferes with occlusion. Keep teeth in the socket B. Community Periodontal Index of Treatment needs 423. Vertical incision of mucoperiosteal flap should be: A. Narrow B. 422. C. 419. Bitewing Periapical Occlusal Panoramic What does CPITN stand for: A. Bisect the middle of gingival papillae C. Which radiographic method would you use in assessing periodontal conditions and lesions: A. False periodontal pocket B. Gingival cleft . Always extending to the alveolar mucoperiosteal B. Flat ridge Faulty or not proper contour Not properly polished restoration Cervical wall is too deeply apical Overextension of lining in cavity Angular type of bone resorption can be seen more often in:** A. Provide nutrition 420. C. B. Apical migration of the epithelial attachment followed by atrophy of marginal gingiva at the same level results in: A. C. 418. E. Protect alveolar bone C. Periodontal pocket recession C. D. The periodontal ligament in a teeth without use appear to be: A. D. Wide 421.A. Must be at the right angle of the tooth 424. D. B. B. Occlusal traumatism Food particles retention Periodontosis All of the above What is the most important function of periodontal ligament: A. The incision angle in Gingivectomy is: A. 426. Calculus attaches to teeth surface by:** A. B. 429. 0. C. Cementoma C. B. C. D. B. Incisive foramen when are superimposed over apex of root on radiograph may be mistaken to be: A. D. Interdental papillae The free gingival ridge The attached gingiva The marginal gingiva Which is the MOST local factor in the aetiology of periodontal disease: A.5mm B. C. True pocket 425. Odontoma 431. The MOST common place for initiation of gingivitis is: A. Chemical irritation and caries Trauma and function Attrition.25 to 0. C. 1mm 427. wear and aging of the patient All of the above . 45° to the tooth in an apical direction 428. E. B. Cyst B. Occlusal trauma Calculus Brushing habits Coarse food 430. Acquired pellicle Interlocking to the crystals of the tooth Penetrated into enamel and dentine Mechanical interlocking All of the above The width of normal periodontal ligament space is: A.D. D. Which of the following factors can affect the shape and size of the pulp canal: A. D. Max Anaesthetic Concentration” B. 433. D. Following a periodontal surgery. Sucrose 436. Probe in mesial distal and mid facial areas of suspected tooth 437. Buffering action 434. How to detect the furcation involvement: A. Lingual nerve is anterior and medial to inferior alveolar nerve 440. 3% prilocaine with felypressin B. It cooperates into plaque and resits acid demineralisation B. Which local anaesthetic agent is preferred for a confirmed hypersensitive patient: A. Mepivacaine 3% without vasoconstrictor ** . N2O has high analgesic property and low anastatic at its minimum anaesthetic dose. C. Which of the following is correct about Nitrous Oxide N2O: A. A patient comes with a lactobacillus of more than 100000.432. what is your advice: A. B. It is good aesthetic and low MAC 439. Which is CORRECT about the Lingual Nerve: A. Has low blood diffusibility and result in hypoxia D. What is TRUE about topical fluoride:** A. Radiolucent area radiographically B. Reduce sugar in diet 435. The MOST cariogenic sugar is: A. Help in tissue adoption Decrease the patient’s discomfort Enhance the rate of healing Control bleeding and maintain blood clot What is the MOST important role of saliva in preventing dental caries: A. Absolutely contraindicated in pregnancy C. periodontal dressing will: A. “Low MAC. Fluoride prophylaxis paste has been clinically proven to be more effective preventing caries 438. Protein denaturation 445. A primary molar with relatively un-resorbed roots encompassing the permanent tooth bud.** A. Soak them in hypochlorite solution “Milton” Sterilize. It kills all pathogens but not spores. 443. Section the tooth vertically and remove each root separately 448. Which is TRUE about disinfectant solution:** A. Tumour . What would be your diagnosis? A. Moist heat sterilization B. It reduces the number of micro organism to a non infective level C. The first thing to do after surgical removal of impacted 3rd molar in the mandible is:** A. All of the following are requirements of an adequate mucosal periosteal flap except:** A. B. What to do with instruments after surgically treating a patient with confirmed diagnosis of hepatitis B. scrub and sterilize Handle them with two pairs of household rubber gloves Scrub them with iodine surgical solution What is the mode of action of autoclaving “Moist sterilisation”: A. Mucous membrane is carefully separated from periosteum C. C. The MOST common side effects of local anaesthetic is a result of: A. Cold application from the outside 447. Intravascular injection B. Base containing blood supply 446. 444. A young female patient presents with throbbing pain in the left lower posterior jaw with trismus and associated lymphadenopathy.441. D. Base is wider than the free margin B. Hypersensitivity 442. It destroys all pathogenic micro organism including high resistant B. What extraction technique would you use to avoid the inadvertent removal of a developing bicuspid A. Referred pain C. Odontalgia B. Moniliasis /Which is candidiasis/ 454. Irritation fibroma 452. has indurated margins. The base is narrow. It has been present for about six months. covered by greyishwhite exudate. the patient is taking this antibiotic for the past two weeks. Which of the following are not supplied by the mandibular division of trigeminal:** A. White skinned people B. A patient is complaining of an open sore on the buccal mucosa. Insisting that you extract his lower teeth. Trigeminal neuralgia 450. You diagnosis is: A. Masseter muscle C.5 cm in diameter. He is concerned because this might represent cancer. The lesion is painless. He admits “playing with it”. 1. the most likely diagnosis is: A. The diagnosis is:** A. An old male presents complaining of having numerous white lesions in the oral cavity within past few days. enlarged lymph nodes and tender. Patient presents to you with a history of local pain in the lower right posterior region. 30 years old male complains of painless swelling in the buccal mucosa. Buccinator 451. ulcerated. How can differentiate between a benign epithelial tumour from a carcinomatous one: . Chancre /Primary lesion of syphilis/ 453. Pericoronitis 449. lesions are relatively non-painful. It reacts far simply to radiotherapy 455.B. Anterior part of digastric B. buccal mucosa and the tongue. The teeth in question are vital without any pathology. Characteristic of Squamous Cell Carcinoma:** A. MOST LIKELY to be: A. negative tuberculin test and positive serology. Alcoholic and smokers C. slightly elevated. adhere plaques on the lip mucosa. Prior to this the family physician prescribed chlorite tetracycline for an upper respiratory infection. Which of the following is true: A. Buccal first to move tooth Palatal first to move tooth Distal first to move tooth Rotation movement Fraction of the tooth 461. Your treatment is: A. C. C.A. High malignant potential B. 8 years old child. not indurated or not fixed /Move freely/ and pedunculated. Extraction of 75 allowing 36 to move mesially B. E. Extraction of 65 and 75 462. Extraction of 75 and place a fixed space retainer to be replaced with fixed bridge. 456. C. Red band on the free gingiva associated with platelet. on examination you find 75 with carious exposure. Soft papillomatous mass. Some malignant potential 459. Pulpotomy on 75 and wait indefinitely C. Antibiotics are useful in the treatment of periodontitis . D. 460. B. What is the characteristic feature of gingivitis in AIDS patient:** A. 457. 458. B. D. B. Correlating with other pathogenesis lesions of AIDS and does not resolve to periodontal conventional treatment. the movement should be: A. Blood metastasis Does not erode bone Intensive involvement / inveterately characteristic/ Radio resistant What is the significance of erosive lichen planus:** A. Sever pain The characteristic feature of basal cell carcinoma is: A. At the root apex At the bifurcation On the buccal side of the tooth On the lingual side of the tooth To extract upper deciduous molar. D. On Xray you find 35 missing. Where does the bone resorption show in a necrosis pulp of deciduous molar: A. C. B. D. B. 465. Antibiotic and analgesic C. Condensation silicone C. D. E. D. Trauma from occlusion causes thickening of the marginal gingivae Periodontitis is the primary cause of teeth lost after the age of 35. C. A. D. Clicking B. Trigeminal nerve Glossopharyngeal Facial nerve Recurrent laryngeal 468. B. B. 464. 463. Open and drain for two days B. Amalgam Gold inlay Composite Glass Ionomer Acute apical abscess-emergency treatment: A. Which impression material should NOT be kept in water within on hour: “in another paper was: 30 mins before pouring” A. Polyether B. Locking C. Polyvinyl silicone 469. Class I Class II Class III Class IV Pin Restoration with which material has the best retention: A. D. Increased compressive strength . A. TMJ dysfunction common symptom is. C. B. High copper amalgam lasts longer than low copper amalgam because of:** A. Clean and Ledermix 466. Pain in the muscles of mastication 467. Gagging reflex is caused by: A. C. C. All periodontal pockets can be identified by x-ray Periodontitis is the most common disease in the oral cavity Longest lasting resin restorations are. Zinc Polycarboxylate C. Does not change for the whole life B. What is TRUE about vertical dimension: A. D. C. A. Teeth are overlapped B. When should pour polyether impression materials: A. which cement will you use: A. GIC 476. B. Increases when a lower denture is placed in mouth 473. There is maximum cuspal interdigitation 472. To compensate for the expansion of investment. The shortest facial height is when: A. 475. The sterilisation of Gutta Percha is achieved by: A. If temporary cementation is required. Parallel side towards the path of placement. Insufficient pressure during flasking causes it 471. guiding planes are created by. In the construction of an RPD. ZOE B. E.B. Perpendicular to the occlusal plane B. Contraction porosity in thickest point of the denture B. D. Increased corrosion resistance High creep Increased tensile strength Decreased setting expansion Porosity in acrylic dentures is caused by. Within 24 hours after taking impression B. A. 474. C. E. Heat Chemical sterilisation Flame Boiling Autoclave Why would you cast gold in hot mould:** A. 477. Decreases when head is tilted back C. Within 30 minutes after taking impression . 470. Your treatment will be: A. Can extend 0. A significant number of female are allergic to chromium C. A significant number of females are allergic to nickel B. The patient requests an aesthetic filling. B. Porcelain layer is too thin over the opaque layer. A gingivally extended chrome cobalt cast clasp: A. B. Same as amalgam with cavo-surface bevels B. Pulpectomy B. Which of the following is a frequent cause of opaqueness in a porcelain jacket crown: A. The pulp horn is visible as a pin point. Will resist deforming forces better compared to cast gold 482.5 under the surveyor line B. Can extend 0. B. Your preparation will be: A. D.C. Proximal caries removal with occlusal & gingival bevels 481. Which cement is less soluble in the oral cavity: A. The bone resorption will be the same for both arches Resorption is more on the palatal side of maxillary molars Resorption is more on lingual side of mandibular molars The ridge height resorbs more in maxilla than mandible 483. C. A significant number of males are allergic to nickel 484. D. Polycarboxylate Zinc phosphate Silicate phosphate GIC 480. Porcelain layer is too thick 479. Should be stored dry and then poured D. C. A seven year old boy fell of his bicycle 2 weeks ago and broke his maxillary central incisor. The first molars are extracted in both arches: A.25 under the surveyor line C. The tooth is vital. Place calcium hydroxide and fill with composite resin . The use of nickel chromium in base plate should be judiciously considered because: A. A patient with reasonable oral hygiene has a small proximal caries on the premolar. Should be stored in humid place 478. Sub micron sized particles C. Mylohyoid B. D. 487. Which muscle contracts during the tongue protrudes: A. Which of the following is the most significant factor regarding filler in composite for increased strength: A. Particle size 1-3 micron B. During mouth preparation for RPD on tooth adjacent to edentulous area. Orbicularis oris 489. Proceed with rest seat preparation and fabrication if involved area is not more than 2mm 486.C. A patient has a small incisal fracture of the maxillary incisor. Genioglossus C. To avoid this what process should have undertaken initially: A. D. There is dentine exposure: A. Restoration is required B. 488. Ask patient to bite firmly while impression is setting Hold the metal base frame against the abutment tooth while setting Fabricate new denture Add impression material and close the gap Enamel Dentine Pulp Cementum The muscle responsible for maintaining the bolus of food while chewing is: A. Which is the only dental tissue that lose its formative cells as it matures: A. C. High concentration of the filler particles 491. After making an impression to reline an RPD the dentist notes that the indirect retainers are not resting on the tooth. Buccinator B. B. Micro-filled composite B. Which is the best material to resist fracture at the acid etched tooth composite interface: A. Calcium hydroxide pulpotomy 485. B. C. Digastric 490. Hybrid composite . C. GIC D. Silicate 492. The principal factor involved in oral Para-function is related to: A. Periods of stress B. Occlusal pre-maturities during mandibular closure 493. During manual palpation, the mucosa is thin in: I. Midline of the palate II. Mylohyoid region III. Over torus palatinus A. B. C. D. 494. I, II and III None of the above I and II II and III The path of condyle during mandibular movements depends on: A. Articular eminence, meniscus/capsule of TMJ and muscle attachments 495. While doing RCT you gave dressing with a paper point wetted with..??...solution. The patient arrives the next day with severe pain. There is no swelling but the tooth is tender to percussion. You will:** A. B. C. D. E. 496. Replace with similar dressing and prescribe antibiotic Replace with corticosteroid past Retrieve paper point surgically Remove the dressing and leave for several days before replacing it. Provide incision and drainage The area of the posterior palatal seal includes which of the following: A. [left][right] B. Hamular notch 497. The best method of cleaning and toilet cavity: A. B. C. D. 498. Alcohol Citric acid Water Organic acid Herpetic infection is an iatrogenic infection spreads by the infected: A. Serum B. Vesicle C. Vesicle fluid and saliva 499. Periapical abscess is differentiated from periodontal abscess by: A. Pulpal radiology B. History and vitality test C. X-ray and history 500. You may suspect poor reaction to bleeding if there is a history of: A. Cirrhosis of liver B. Hypertension 501. Maxillary central incisor located palatally causes : A. Prolong stay of primary central incisor B. Supernumerary teeth 502. Toxicity of anaesthetic is assessed by: A. Dose which is given B. Percentage of solution C. Vasoconstrictions amount 503. Children born with cleft palate, microdontia and glossoptosis have: A. Christian disease B. Trenches-Collins Syndrome C. Pierre-Robin Syndrome 504. Which of the following penicillin are readily destructed by stomach acid: A. B. C. D. 505. What is not correct about Long Buccal Nerve:** A. B. C. D. 506. Methicillin Cloxacillin Phenoxy methyl Penicillin G Passes through two heads of pterygoids muscles Supplies mucosa over lower and upper molars Supplies the buccinator muscle Supplies skin over buccinator N2O excretes through: A. Urine B. Lungs 507. Radiopaque lesions are seen in: A. B. C. D. Multiple myeloma Paget’s disease Hyperparathyroidism Chronic renal failure 508. The causative organism in localised juvenile periodontitis is Actinomyces actinomycete comitans which is:** A. Gram positive facultative aerobic B. Gram positive facultative anaerobic non-motile rod C. Gram negative facultative anaerobic non-motile 509. Which of the following is NOT significant factor in determining virulence of a.a.??** //I think a.a. stands for Anaerobic Bacteria/ A. B. C. D. 510. It effects chemotaxis Produces leukous toxins Destroys collagen It is immuno-suppressive Density of film is decreased by increasing the : A. B. C. D. 511. MA Exposure time Developing time Rinsing time The best space maintainer is: A. Lingual holding arch B. Pulpectomised primary tooth C. Band and loop maintainer 512. The laboratory findings in Paget’s disease show: A. B. C. D. E. 513. Elevated calcium, elevated phosphate, and elevated alkaline phosphate. Normal calcium, normal phosphate and elevated alkaline phosphate Decreased calcium, increased phosphate and elevated alkaline phosphate Increased calcium, normal phosphate and decreased alkaline phosphate Normal calcium, increased phosphate and elevated alkaline phosphate While giving CPR which of the following is considered: A. It achieves 30% of cardiac output with 60 compressions per minute D. C. normal enamel Enamel is missing but dentine formation is normal Enamel and dentine show disturbances Pulp is normal but dentine is abnormal A 10 year old boy presents with non-vital. B. Pulpectomy with calcium hydroxide Pulpectomy with Zinc oxide eugenol Pulpotomy with formocresol No treatment is required if tooth is asymptomatic 519. B. You have to check compression point by thumping before starting compression D. E. A patient suffers a blow to his maxillary central incisor without resulting in fracture. Small pulp chambers and root canals. Delayed eruptions of at least part of the dentition is a recognised feature in:** A. Rickets 517. 518.B. non-mobile tooth. C. B. Your initial response is: A. Administer glycerine trinitrate and monitor patient in upright position Patient has an acute episode of angina as demonstrated by curve in ECG No treatment is required until confirmed as MI by ECG Patient has myocardial infarction as confirmed by ECG 515. D. which structure would you expect to find: A. 514. Anhidrotic ectodermal dysplasia C. A patient has developed a sever chest pain and difficulties in breathing while in the dental chair. Treatment is: A. On inspection of lateral boarder of the tongue at the base. Immediate necrosis B. C. The pulp: A. Cardiac output has to be monitored regularly by checking radial pulse. D. Becomes non-vital but only if treatment is delayed too long . It achieves normal blood oxygen levels with 12 reseparations per minute C. Filiform papillae Fungiform papillae Taste buds Lymph nodes Circumvallate papillae 516. B. Dentino-Genesis imperfecta B. C. D. Which of the following is a radiographic feature of dentino-genesis imperfecta: A. Eliminate sugar from diet 525. 523. Have the patient to fill in a diet survey C. In the case f malignant melanoma occurring intra orally. B. C. as this will increase metasis The 5 years survival rate is 20% The incidence of oral melanoma is the same as those on the skin Commonly occurs intra orally 521. D. Post operative bleeding can be reduced somehow by using tranexemic acid Prothrombin values of at least 2.5 is required to perform extraction It takes at least 8 hours for heparin to take affects Heparin should be administered sub-cutaneous 90-95% reduction of caries 45-55% reduction of caries Reduces pit and fissures caries more than smooth surfaces Reduces smooth surfaces more than pit and fissures Patient presents with caries in many teeth. E. Fluoride toothpaste does not effectively prevent caries and topical fluoride is required. Nervousness . Becomes non vital irrespective of treatment D. Secure details of patient’s eating habits B. Which of the following is NOT true:** A. Which of the following is true in regards to periapical cementoma: A. Teeth are not vital 526. C. D. C. What is the primary consideration in providing nutrition/dietary counselling to a patient: A. Teeth are vital. B. B. you will advise that: A. Community water fluoridation MOST effectively achieves is:** A. Uncommon on the palate when occurs intra orally Should not biopsied. Patient on anti-coagulant therapy requires an extraction to be performed. which of the following is true: A. B. 522.C. Angioneurotic oedema B. No changes is seen later if fracture does not occur 520. Which of the following is not a side effects of lignocaine: A. D. 524. the patient develops paralysis of eyelid. D. Ansa cervicalis C. Two lateral surfaces Lateral surface and face Lateral surface. face. back and shank Which of the following is NOT TRUE in regards to lateral periodontal cyst** A. D. When you focus your light into his eyes he turns away. The disease and lesions are: A. Damage/injury to which nerve causes dilation of pupils: A. The boy has a fever of 40°C and coughing. C. A physician refers a nine year old boy to you to confirm diagnosis. The parotid gland . After an inferior alveolar nerve block. D. C. This means that the L. derived from the remnants of the dental lamina 530. Abducens 533. Situated on the anterior boarder of sternocleidomastoid muscle 532. the blade is formed by which two surfaces: A. Calculus in the salivary duct resulting in sialolithiasis. B. Ranula Cyst Mucocele What is TRUE in regards to branchial cyst: A. A. In periodontal scalers and curettes.527. face and shank Lateral surface. Middles aged woman gives a history of intermittent unilateral pain in the sub mandibular region. 531. B. C. 529. upper lip and lower lip on that side. AHGS vesicles 528. Measles and Koplik’s spots B. Intra-orally there are white spots surrounded by red margins. most probable cause is. Oculomotor B. B.A was deposited in: A. It is more common in anterior region It occurs more in maxilla than mandible Probable origin is from dentigerous cyst which develops laterally Encountered in the cuspid-premolar region of the mandible. I. It blocks the cyclo-oxygenase pathway. First proximal increment was too large III. II. III II.534. Odontoblast is drawn into the tubule 539. Insufficient condensation II. Neglecting to wedge the matrix IV. 535. In RCT the ideal root filling: A. Perpendicular to long axis C. III. Increased prothrombin time B. V None of the above All of the above 538. C. Extends beyond apex to achieve a good seal C. Patient with haemophilia presents which of the following findings:** A. It is anti inflammatory by the release of histamine B. Debris contamination A. The pulpal floor of the Class II cavity for a mandibular first premolar should be:** A. E. II. Increased clotting time 536. Aspirin reduces pain by which of the following mechanism: A. Hand manipulation instead of mechanical V. B. Parallel to occlusal plane B. Increased bleeding time C. Ends at the dentino-cemental junction 540. At the radiographic apex . D. Where is the narrowest part of the pulp: A. Marginal leakage at the proximal gingival cavosurface of a recently restored class II can be caused by: I. Tilted lingually 537. Ends at the apex B. What are the dangers of using air as a cooler during cavity cutting: A. Hypersensitivity B. IV I. Deformity opposed the applied load 546. An internal force to oppose external load C. erosion. 0. Self polymerising acrylic resins differs from heat cured resins because they exhibit: . Which of the following is MOST useful in differentiating between apical abscess and periodontal: A. Percussion Vitality tests Cold tests Heat tests 542. It should be selected before starting preparation Chroma is the lightness/darkness of colours Value is the colour itself Hue is the concentration of colours How many mg of fluoride ions are obtained from 2. 543.5 times that of the crown Same as the anticipated crown Which is correct in regards to shade selection of crowns: A. C. C. D. C. C. B. B. At the dentino-enamel junction C. What is the ideal length for a post in post-core in an endodontically treated tooth: A.B. D. Age Parafunctional History of the tooth /abrasion. D. 545. Size of pulp chamber within the tooth is influenced by: A. caries/ All of the above 547. 544. D. C. An external force B.5mg 1 mg 1. D. B. At the orifices 541.5mg 10mg Strain is defined as :** A. 2/3 of the tooth length ½ of the tooth length 1. B. B.2 mg tablet of NaF A. Mandibular lingual in the midline C. 20% have 2 canals with one foramen 20% have 2 canals with two foramina 40% have two canals with 10% ending in two foramina 40% have two canals with only one ending in two foramina Splinting the adjacent teeth in fixed bridge is primarily done to: A. In lingual pits B. C. In buccal fissures C. 554. Where would you expect to find the Mylohyoid on relation to periphery of complete denture: A. Mandibular buccal in the midline B. Achieve better retention . The contraction (Gaseous) porosity in inlays is related to:** A. Overheating of the alloy Molten gases Diameter of the sprue Overheating of investment 550. B. Buccal and lingual undercuts 553. Significantly lowers firing temperature 549. Removes water before firing. Retention for occlusal amalgam cavity in premolars is BEST provided by A. Mandibular disto buccal area 551. increasing the hardness of porcelain C. Distribute the occlusal load B. Class V lesion may originate: A. Higher content of residual monomers 548. B. The advantage of firing porcelain in vacuum:** A. Reduces size of air-bubbles incorporated thus decreasing porosity B. D. What is true in regards to lateral mandibular incisor A. Higher molecules weight B. Poor oral hygiene 552. Mesial and distal undercuts C. D. Slightly undercutting of walls with inversed cone bur B. C.A. Occlusal /Above/ C. Pre-soldering and heat treatment Heat treatment and opaque /bake/ stages Opaque and bisque stages Bisque and glazing stages First opaque bake and second opaque bake 556. Which of the following liquids is not suitable for prolonged immersion of cobalt chrome partial dentures: A. What is the relationship of the retentive portion of the partial dentures retainers to the survey line of abutment: A. Reduced Overjet of posterior B. Increases when lower denture is inserted C. 558. No relation 557. C. In complete dentures. Increased vertical dimension C.555. 562. D. Resting face height in edentulous patients: A. Porcelain must not be contaminated by handling between which two stages: A. C. D. Gingival /Below/ B. B. Alkaline peroxidase Hypochlorite solutions Soap solutions Water Dentures hyperplasia is generally attributed to: A. Ala-Tragal line is:** A. Poor oral hygiene B. C. Decision to employ cusped or without cusps teeth is influenced by:** . cheek biting is most likely a result of: A. Decreases when head is tilted back B. E. B. B. Denture movement 559. The line running from the tragus of the nose to ala of the ear A guide used to orient the occlusal plane Parallel to Frankfurt horizontal plane A guide to the occluding face height in complete denture. Does not change over time 561. D. Teeth have large cusp inclines 560. High in content (High filler) 566. TMJ problems C. Precipitated from carbohydrate C. The first forming microbial elements of plaque are: A. Bacteria from sucrose B. Sensitivity to hot and cold foods soon after cavity preparation and placement of GIC and composite resin in an upper incisor tooth is due to: A. Sub micro sized particles B. Cranio mandibular skeletal relationship 563. Heat from GIC settings 567. Over instrumentation extending into periapical area Irritation from chemicals used Entrapped bacteria One or any combination of the above It is desirable to major connectors of upper partial dentures to:** A. Aerobic gram positive G+ Aerobic gram negative GAnaerobic gram negative GSpirochetes Anaerobic gram positive G+ Extracellular polysaccharides in plaque are formed by: A. 568. 564. High coefficient of thermal expansion C. C. Precipitated from glycoproteins 565. Chemical C. debridement and placing a dressing. apical periodontitis is because: A. C. B. Mechanical trauma due to cavity preparation B. What is important requisite for fillers in dental composite restorative resins in load bearing area: A. Reverse Overjet B. B. At least 5 mm away from the gingival margin B. D. Cover the anterior palate 569. E. Which of the following statements is true: . After completing pulp extirpation. D.A. 3mm at the margin. B. D. Down’s syndrome B. D. nauseas. you will: A. E.A. D. 571. Xerostomia Mucositis Increased caries Heightened taste sensation Increased risk of osteomyelitis 574. D. Which of the following is NOT complication of radiation to head and neck area:** A. When you tries to seat a crown on tooth you find a discrepancy of 0. Decreases blood pressure and causes headache 573. B. 572. C. A female patient is diagnosed with Addison’s disease which of the following does not confirm this: A. Steven Johnson’s syndrome . fatigue Hypotension Bony expansion Amenorrhea 575. Which of the following conditions is not associated with periodontal destruction in primary teeth: A. Last secretion of odontoblast forms cementum Last secretion of odontoblast forms acquired enamel cuticle Remnants of ameloblasts form primary enamel cuticle Remnants of odontoblasts form primary enamel cuticle 570. B. Reduce inner surface of crown Remake a new crown Smooth the enamel at the margin Hand burnish crown margins In regards to Chlorhexidine mouth wash:** A. B. C. C. C. Gives relief of pain by decreasing venous return B. B. E. Is anionic Used in 0. D.02% concentration Used in 0. Weakness.12 concentration Penetrates the gingival crevice/pocket Glycerine trinitrate given to an angina patient acts by: A. lassitude Anorexia. C. Papillon-Lefebvre syndrome E. You will:** A. Cariogenicity of Streptococcus mutans is because of the production of: A. Patient is resistant to caries but has periodontal disease. D.C. It is because of dental hypersensitivity 577. Ask to use low abrasive dentifrices B. E. Glucans Levans Fructans Sucrose A child consumes a toxic dose of fluoride. B. In this case. Reduces the size of the beam. sucrose in diet is important because: A. Sucrose is greatly involved in plaque development S. so it is easy to visualise the central X ray. It reduces the errors in gathered data. mutans produces Levans frictions which are used by periodontal pathogens The streptococcus mutans cannot survive with a continual supply of sucrose Existing plaque must continue to get sucrose in order to grow Induce vomiting Gives a lot of fluids Gives a lot of fluids and sodium bicarbonates Ask patient not to eat for 45 minutes Gives milk. D. calcium tablets or magnesium tablets Collimation is done to: A. Cyclic neutropenia 576. 579. 582. B. 578. C. 580. B. calibration of examiners data is important because: A. Avoids unnecessary exposure to radiation of surrounding tissues of the patient . D. 581. D. Gingival index by Loe and Silness Periodontal index Periodontal disease index OHI-S In surveying. B. In patient with exposed root surfaces: A. C. C. B. Which of the following is the best index to evaluate gingival health: A. C. Hypophosphatasia D. the epitaxic concept is one of the theories. Mineralisation occurs when calcium and phosphate content is high B. 587. The amorphous materials would convert to calcium phosphate and hydroxy phosphate 585. Fixes the developed film 589. D. Removes exposed silver halide C. twining B. the outcome film will be: A. Removes unexposed silver halide crystals B. Orient exposure side B. Reduce exposure time C. C. Fogged 590. Fusion of two or more crowns of teeth C. Reduce size of the beam 584. In primary teeth. In X rays filtration is used to:** A. Fusion of two or more roots 586.583. but the solution is too warm. Kaposi’s sarcoma: . Remove low energy X rays B. What does the fixes solution in developing X rays do: A. External resorption Internal resorption Necrosis of the pulp Ankylosis A raised dot on X ray film is present to: A. Too dark C. Gemination is:** A. B. Differentiate between left and right side C. The presence of matrix would start initiate formation of nucleus C. failure of Ca(OH)2 pulpotomy is MOST likely to produce:** A. Which of the following is true: A. Division of single tooth. When the developing solution is correctly mixed and x ray film is being developed for normal time. Dip during developing 588. In calculus formatio. Too light B. A. Seen on buccal mucosa in HIV as purple lesion B. Seen on palate of most HIV patient C. Should be biopsy 591. What is characteristic feature seen in pyloric stenosis:** A. B. C. D. 592. Erosion of maxillary central incisors Vomiting of undigested food Loss of appetite Weakness At birth, the oral cavity usually contains: A. B. C. D. S. mutans only No micro organism S. mutans and S. salivavis Lactobacilli and S. mutans 593. The papillae that are few in numbers, associated with MOST taste buds, associated with Von Ebner’s glands are:** A. B. C. D. Fungiform Circumvallate Foliate Filiform 594. In class II preparation it is difficult to place the gingival seat when preparation is extended too gingivally because the: A. Enamel rods are directed occlusally B. Marked cervical constriction 595. In maxillo fracture, if intra cranial pressure increases:** A. B. C. D. E. 596. It is normal Typically associated with tachycardia Associated with blood pressure Usually subsides spontaneously Typically associated with constricted and un-reactive pupil Moist heat sterilization is achieved by: A. Denaturation of protein 597. In regards to Benzodiazepines: A. Increases R.E.M. sleep B. Has a hangover effects because of active metabolism C. Includes carbamazepine D. Can be used safely on children as it achieves reliable effects 598. Which is NOT CORRECT in regards to lingual nerve: A. It is posterior and medial to the inferior alveolar nerve B. It passes close to the mandibular 3rd molar C. It may be anaesthetised by the mandibular nerve block D. It provides supply to the lingual gingiva E. Supplies anterior 2/3 of the tongue 599. The maxillary and mandibular teeth get their blood supply from:** A. Separate branches of S. Palatina artery B. Separate branches of maxillary artery C. Branches of maxillary and mandibular arteries 600. Haemophilia is characterised with:** A. B. C. D. E. 601. Daughters affected from their carriers fathers Presents on “y” chromosome Hemarthrosis is common finding Deficiency of factor VII Neutrophil defect In minor oral surgery which is TRUE in regards to antibiotic: A. Amoxil is satisfactory against most oral infection B. Metronidazole and Amoxil have the same penetrating power C. It is evident that it will reduce post operative swelling 602. In regards to third molars surgery: A. Maximum swelling is seen after 24-48 hours B. Prophylactic antibiotic will reduce swelling C. Antibiotic cover is compulsory 603. A 65 year old patient needs extraction of 44; he has taken insulin in the morning. What preoperative advice you should give: A. B. C. D. Take more sugar Maintain normal diet Antibiotic 2 hours before Medication increases preoperatively 604. Patient with prosthetic heart valve taking 7.5 mg warfarin. She has????. Patient needs extraction. What is your management: A. B. C. D. E. 605. 3g Amoxil, suture after surgical removal 3g Amoxil, suture when bleeding has stopped Gentamycin/vancomycin cover, stop warfarin, give heparin and suture later Ampicillin cover, stop warfarin, give heparin and suture later Gentamycin/vancomycin cover, stop warfarin and suture later Loss of the gingival attachment is measured between: A. CEJ to base of pocket B. Top of the gingiva to the base 606. Absence of clearly defined crystal lamina dura is because: A. B. C. D. Pathognomonic of periodontal disease Indicative of attachment loss Associated with periodontal pocket Commonly related to radiograph angulation 607. A patient 37 year old; with paroxysmal pain on the left eye that he thinks is related to his maxillary posterior teeth. The pain comes in recurrent bursts and aggravated by stress and alcohol. Oral exam is negative. The probable diagnosis is: A. B. C. D. 608. A mandibular permanent first molar had to be extracted, this will affect: A. B. C. D. 609. Migraine Cluster headache Trigeminal neuralgia Temporal neuritis Adjacent teeth Teeth in the same quadrant Both arches the same side Full mouth The places for new erupted mandibular molars are created by: A. Resorption of anterior ramus and apposition posteriorly B. Apposition of alveolar process C. Apposition of inferior boarder of mandible 610. A patient comes with a firm, painless swelling of lower lobe of parotid which has grown progressively for the past year. He complains of paresthesia for the past 2 weeks. This is most likely to be: A. Pleomorphic adenoma B. Carcinoma of the parotid C. Lymphoma of parotid What is the histopathology of the pathogenesis of the plaque following 21 days of plaque accumulate: A. Removes required pellicle . Picture of ANUG superimposed with RPP Spontaneous bleeding interproximal Depression of T4/T8 lymphocytes Deep Perio-pockets usually seen in advanced periodontitis What is true in treating a patient with secondary herpes simplex:** A. C. Fluorosis B. B. Oral prophylaxis. D. D. Oral prophylaxis and gingivoplasty B. Antivirals are contra indicated in immuno-compromised patient 614. Acyclovir inhibits viral transcription when applied in the prodromal phase B.611. Which is wrong in regards to (water jet spray) hydrotherapy: A. Stop medication 616. Primarily infiltrate of plasma cells Primarily infiltrate of lymphocytes Infiltrate of plasma cells and early bone involvement Infiltrate of neutrophils What is INCORRECT in HIV associated periodontitis:** A. 613. Removes plaque C. reduction in size of pulp chamber. scaling. Dentinogenesis imperfecta 617. The MOST common cause of gingival enlargement is: A. 612. C. A patient has improperly formed DEJ. chipping and attrition of enamel that would MOSTLY be: A. D. Does not harm gingivae B. root planning C. Hereditary Drug induced Plaque induced Leukaemia 615. gives a history of Dilantin sodium what is you treatment: A. Amelogenesis imperfecta C. Idoxuridine is better than acyclovir when applied topically C. A 13 year old has enlarged gingivae. B. B. C. What is the MOST common consequence of an allergic response to medication: A. Which of the following procedures should be employed:** A. Calm down the patient 625. What would you do if the systole is elevated:** A. Remove the tuberosity and suture Leave the tuberosity and stabilize if required Remove the tuberosity and fill the defect with Gelfoam then suture. What cause a reduce of pulmonary ventilation: A. Surgically excise the entire lesion since you know it is not malignant C. it remains in place attached to the mucoperiosteum. Inform the patient and her physician of your findings and instruct the patient to return in six months B.618. An incision biopsy of an ulcerated and intruded clinically suspicious lesion in 50 years old female reveals chronic inflammation. the tuberosity is fractured. Dismiss the patient with instructions for warm saline rinses for re-examination D. During extraction of maxillary third molar. Laryngeal muscle paralysis B. Investigate systemic cause 626. Air way obstruction 624. Skin rash “dermatitis” with swelling of lips and eyes 622. Hypodontia or anodontia 619. D. B. 4-6 times a minute 623. however. What would you do if the diastole is elevated: A. How many time do you breath in mouth to mouth resuscitation: A. Repeat the biopsy 621. you would:** A. C. Which are non-calcified areas in the child’s cranium: . 10-12 times a minute B. If fractured tuberosity is greater than 2 cm. Anhidrotic ectodermal dysplasia is characteristic by:** A. leave in place and suture 620. 628. C. Bacterial endocarditis 634. D. Whooping cough C. D. 6000000 units of benzoyl penicillin B. How do treat the cause of airway obstruction: A. Viral infection Diabetes Measles Rubella Candidosis What is Von Reckling hausen disease: A. Antibiotic therapy Poor surgical techniques Aspirin Codeine Acute pyogenic bacteria infection may result in:** A. Fontanelles 627. Necrosis of bone produced by ionizing radiation 629. E. Which is LEAST likely to cause bleeding after surgical operation: A. Herpes simplex B.A. Neurofibroma B. Prophylactic administration of antibiotic is indicated in patient before oral surgery with: A. Extension of the neck B. C. D. 2g Amoxicillin pre-operatively 631. Koplik’s spots are associated with one of the following: A. Leucopoenia Neutropenia Leukocytosis Lymphocytosis Eosinophilia 633. C. E. B. Oral mucosa and skin pigmentation occurs in patient with:** . Flexion of the neck 630. How do prepare a patient with rheumatic fever before extraction: A. B. 632. B. Antibiotics are useful in the treatment of ANUG B. E. 639. Perio disease is a primary cause of loss of teeth after 35 years of age. D. C. E. Oedema B. Trauma of occlusal factors causes cleft or fibrous thickening of marginal gingivae C. All Perio pockets can be detected by x rays D. Which of the following is TRUE: A. B. B. D. Polycythemia 638. the signs are. D. In regards to the conditions where you have to prescribe antibiotic prior to dental treatment: A. shallow respiration. F. C. B. Rheumatic fever Sub-acute bacterial endocarditis By pass Valve replacement Uncontrolled diabetes All of the above .A. blanched face. Leucopoenia B. your first management is: A. Patient has fainted. supine Thrombo cytopenic purpura would complicate surgery by: A. moist skin. Leukaemia C. Patient who has WBC count of just over 100000 is most likely suffering from:** A. weak pulse. F. Haemorrhage C. Periodontitis is the most common problem in teenage E. 636. Acute infection 637. 1 ml adrenaline subcutaneously Mouth to mouth respiration Nitro glycerine sub lingually Recumbent position. Diabetes mellitus Addison’s disease Multiple myeloma Squamous cell carcinoma Bright’s disease Cushing’s disease 635. C. F. B. Vitamin K 642. Disorder of steroid will result in: A. 643. H. Adrenal suppression Delayed healing Osteoporosis All of the above 647. D. Herpangina is caused by: A. A young patient presented with rheumatic fever and suspected allergy to penicillin.640. Esophagitis. The immediate concern in the management of facial trauma should be: A. Adrenal suppression 646. The antibiotic of choice is: A. 645. E. colitis during 5 weeks. Excitement Shock Improper eye sight Leaning Sever headache Vomiting Euphonia Fixed dilated pupils 644. D. B. B. E. D. G. herpes simplex. The main vitamin to synthesis prothrombin is: A. Chloromycetin Sulphonamide Buffered penicillin Erythromycin Achromycin Patient under treatment with corticosteroids may develop: A. Securing a blood units to replace any loss Fixation of fractures Checking the breath and insure a free airways Neurological consultation What is NOT A SIGN of neurological trauma: A. D. C. Coxsackie virus 641. C. You will find the same signs of: . C. B. C. B. Steam under pressure sterilisation is the best method to kill microorganisms. Dehydration of DNA 651. B. Patient with morphine coma. Shorter C. act: A. They produce penicillinase. Coagulation of plasma protein B.A. Defective formation of clavicles Delayed closure of fontanelles Retention of maxilla Delayed eruption of permanent teeth None of the above 649. D. What does not show in Cleidocranial dysplasia:** A. 652. C. AIDS 648. Near the same size 654. C. D. In regards to Plummer-Vincent syndrome or “Paterson and Kelly syndrome”: A. E. Erythema multiforme C. Longer B. How do you diagnose trigeminal neuralgia MOST accurately: . Multiple myeloma B. C. E. B. 653. How does it work: A. D. we will find the deciduous tooth is: A. Iron deficiency is a feature Atrophic oral and gastric mucosa Dysphagia and angular cheilitis Predisposing oral cancer All of the above 650. When comparing the mesio distal length of second deciduous molar with the length of 2nd premolar. what is the medication of choice to reverse its Bradykinin Epinephrine Amphetamine Naloxone Why are streptococci resistant to penicillin:** A. Ankyloglossia is caused by: A. Symptomatic treatment and acyclovir B. C. Painless bluish lump filled with fluid on the lips. D. C. D. MOST likely is: A. Idoxuridine 658. B. E. E. Like a haemophilic child 656. Like a diabetic child C. B. 659. B. Ameloblastoma on x-rays shows as: A. Mosaic pattern 661. Like a normal child B. D. History 655. Smoker’s keratosis Squamous cell carcinoma Mucocele Fibroma Fibro-epithelial polyp Tzanck cells Test dose of corticosteroid Test of anti body Histological immunofluorescence Serological test for auto antibody Paget’s disease under microscope shows: A. The diagnosis of pemphigus vulgaris is confirmed by:** A. The zygomatic process serves as: A. 657. Origin of masseter muscle Origin of temporalis Protects parotid gland Insertion of lateral pterygoid Treatment of patient with herpes simplex:** A. Soap bubbles 662. How do you treat a child with severe Von Willebrand’s disease:** A. Edentulous ridge . C. 660.A. 670. B. Most congenitally missing teeth are: . Blow to the mandible resulted in deviation to the left on opening. In regards to dentinogenesis imperfecta on x-rays. Osteoporosis B. thin dentine and normal enamel Type III. Short and blunted roots The pulp canal is obliterated Big pulp chamber. Herpes simplex infection 668. C. E. What is NOT CHARACTERISTIC finding in carcinoma of the mouth: A.B. Treatment of Anaphylactic shock: A. The treatment of angioneurotic oedema: A. x-rays show unilateral fracture. Elevation Fixation Invasion Verrucoid appearance Pain 664. D. characteristic shell teeth All of the above Exfoliative cytology will not help in the diagnosis of: A. Anti histamine 10mg IV Chlorphenamine maleate as Piriton by Allen Hydrochloride 25 mg IM Corticosteroid drugs or with adrenaline. C. B. C. D. 667. C. What is TRUE: A. Adrenalin 1mp IV 669. 665. D. Neck of the left condyle Neck of the right condyle Body of the left condyle Body or the right condyle Marble bone disorder is:** A. B. where would you expect the fracture:** A. E. Osteopetrosis 666. Short labial frenum 663. Short lingual frenum C. B. D. D. B. 673. the intensity of radiation is reduced by: A. 675. C. Reduced enamel epithelium of tooth crown Dental lamina dura Epithelium trapped after sutures Hertwig’s root sheath The definition of Leeway space is: A. C. B. Maxillary lateral incisor 671. 677. Pseudo membrane deficiency Herpes simplex Squamous cell carcinoma Elevated Epstein bar viruses incidence Odontogenic cyst develop from the following structures except: A. 5 674. E and 3. C. Mandibular 3rd molars B. Mandibular 2nd premolars C. C. D. 672. D. Preservation of pulp Immobilisation Root canal treatment Calcium hydroxide treatment Healthy dental pulp responds to injury by: A. Vitamin C 676. The formation of reparative dentine at the pulpal surface corresponding to area of irritation . B. The initial priority in treatment of horizontal fracture is: A. D. Which of the following is secondary to immune deficiency: A. B. ½ ¼ 1/3 1/5 Which vitamin is not produced and stored in organisms: A.A. D. 4. If the focal spot to film distance is increased from 20cm to 40cm. It is the difference in mandibular width between C. 25mg 0. Spread of infection 12. Which drug may cause respiratory depression: A. 680. In full dentures. 0. porosity in the most thickest area is due to:** A. interstitial keratitis and nerve deafness in children with congenital syphilis. B. D.00mg Heart disease Asthma Mental retardant Sickle cell anaemia Green stain on tooth surface is due to:** A. 685. Nitrous Oxide in contraindicated in: A. Barbiturate 682. C. 22 is MOST LIKELY to be: A. B. 684. 683. Combination of Hutchinson’s teeth. Gaseous porosity B. What is Hutchinsonian triad: A. Shrinkage porosity 679.678. Delayed expansion Inadequate depth at the isthmus area Inadequate width at the isthmus area Moisture contamination of the amalgam during placement The definition of incompetent lips is: A. Chromogenic bacteria 686. For a 5 years old child who lives in a NON WATER FLUORIDATED are. The most common cause of fracture at the isthmus of class II dental amalgam restoration is: A. D. What is the recommended intake of fluoride: A. C.50mg 1. C. B. Labial .10mg 0. Lips can not close in rest position 681. D. Usually the comparison is between an experimental drug and a placebo or standard comparison treatment. 690. 8 years 11 years 13 years 14 years +2 -2 +8 -8 In hairy tongue you will find: A. What the age of patient who has all incisors. D. After 4 to 7 days. Patient with Class II div I malocclusion has ANB of: A. note that no 2nd molars showing: A. 692. D. B. A kind of clinical study in which neither the participants nor the person administering treatment know which treatment any particular subject is receiving. Sinus tract is indication of: A.B. What does “DOUBLE BLIND” mean: A. Plasma cells 691. Chronic lesion 688. B. C. Pleomorphic oedema 689. 693. This method is believed to achieve the most accuracy because neither the doctor nor the patient can affect the observed results with their psychological bias. what type of cells you would find predominately in gingivitis: A. Superior constrictor of the pharynx B. Palatal 687. Middle constrictor of the pharynx . The MOST common tumour of the parotid is: A. some premolars and some canine erupted. C. Elongated filiform papillae 694. Leukocytes B. Which muscle has insertion in the pterygoid raphe:** A. The first thing to do when syncope occurs in apprehensive patient: A. Between 15 and 25 years of age 702. A non-specific index of inflammation 697. Aerobes B.25mg B. The water fluoridation is 0.00mg . A test that measures the rate at which red blood cells settle through a column of liquid. what is the recommended supplemental fluoride concentrations for 3 year old child: A. C. 0. Dental awareness of the community B. Observation. mostly it bursts spontaneously 701. Scarlet fever Pericarditis Pancreatitis Carbuncle What is TRUE about Chrome-Cobalt partial denture: A. Which of the following is staphylococcal infection: A. Institution of oral hygiene measures C.50mg C. 0. Which of the following has proven to be the MOST important in community preventive program: A. Water fluoridation 703. 1.C. Inferior constrictor of the pharynx 695. Head should be lowered 698. The expected age of patient with rapid progressive periodontitis: A. Aerobes to mainly anaerobes 696. No immersion of dentures in hypochlorite 700. Patient with eruption cyst. Which micro-organisms in periapical lesion you would find microscopically: A.5ppm. D. your treatment would be: A. What is ESR? “erythrocyte sedimentation rate” A. B. 699. What is TRUE in this regard: A.D. 711. Electrons 707. Every year after parent’s permission 710. To reduce the salivary secretion 713. Old expired film 706. what sort of bacteria you expect to find: A. Transcription 709. The pregnancy enlargement of gingivae is a result of: A. To obtain the MOST accurate X rays of teeth. 0mg 704. Every visit routinely B. Periodontitis is usually severe in patient with: A. The transmission of RNA into DNA called: A. Candidal infection 705. The image of x ray is too pale the MAIN cause is: A. Pathogenic means: A. How often a bitewing should be taken for children: A. the tooth film distance should be (Close/far) as anatomical restriction will permit. The beam that goes from cathode to anode is consisted of: A. In the mouth of new born baby. Hormonal disturbance 712. Pathological conditions of the disease 714. Defective neutrophils . The paralleling technique favours the bisecting technique. Why do you give atropine in general:** A. None 708. Rhomboid glossitis is: A. Which of the following conditions would be considered for antibiotic prophylaxes: A. C. Halothane anaesthetic by: A. Extraction of tooth with acute dento alveolar abscess Necrotic ulcerative gingivitis (NUG) unless it is acute.715. Class III cavity is : . Which of the following could cause the overall cellular damage to be greater: A. D. Malignancy recently removed B. Decrease in the tendency to bleed on probing 722. Hepatotoxic reaction 716. Functional heart murmur 720. In regards to Metronidazole:** A. Which lymph node is involved in carcinoma of the lip: “or the first metastasis of carcinoma of lips” A. It is effective for the treatment of AUG/NUG 723. Diazepam 717. How it produces its action 724. The mode of act of drug may be defined as: A. Submandibular node 718. The tissue response to oral hygiene after periodontal treatment is BEST assessed by: A. Sedation in children can be achieved by: A. Submental node B. B. All of the following should be considered for systemic antibiotic except: A. Congenital valve heart disease C. Extraction of 38 or 48 with acute pericoronitis Full mouth extraction for a patient with perio disease 721. The specified dose delivered all at once B. The same fatal dose given in divided smaller doses over a period of time 719. Face bow B.A. Alveolar bone resorption is not seen in: A. Terminal Hinge Axis can be obtained by: A. Remove dental plaque C. Mottling in permanent premolars only 731. Steven-Johnson syndrome (Erythema multiforme) 728. Is thicker that the incisal part B. The silver bromide crystals in x rays films after being expressed to radiation forms: A. Nasmyth’s membrane 733. Palpate the lymph nodes . Has dentine background 727. PA skull x ray B. Hydrotherapy “Water Jet” is used to: A. Articulator 726. Incisal colour differs from gingival colour in that the gingival part: A. Remove pellicle from tooth surface B. you expect to: A. Latent image 729. Kinematic face bow C. Proximal cavity slightly gingival to the contact area 725. No mottling B. The best radiograph for maxillary sinus is: A. Causes no harm to gingiva 732. Occipitomental radiograph C. Mottling in almost all permanent teeth except some molars C. Fluoride in water community of 4ppm will result in: A. Which of the following is not considered in the estimation of gingival index: A. Town’s view 730. When examining intra orally between the side of the tongue and the lateral border of the mandible. Palpate the borders of the tongue 734. D. C. HIV patient 735. Black hairy tongue is MOSTLY seen in: A. Dentinogeneses imperfecta develops in: A. 739. the muscle responsible for elevation of condyle is: A. Employ as much water as possible on mixing Employ as little water as possible on mixing Adding 2% of borax to the mix Adding calcium tetraborate None of the above When dry cast is immersed in water saturated with calcium sulphate:** A. D. 740. What is TRUE about water fluoridation: A. C. There is contraction There is negligible expansion There is definite expansion There is no change None of the above . Will have no effects after the eruption of permanent teeth 736. Masseter muscle 737. C. Initial stage Proliferation stage Histodifferentiation stage Morphology stage Compared to dental plaster all die stones: A. Lateral pterygoid muscle B. Require less gauging water Require more gauging water Require the same quantity of gauging water Are beta-hemihydrate None of the above The MOST effective manner to produce a hard surface on a cast is by: A. E. 738. D. D. E. C. B. B. E. B. When there is a fracture on condyle. Medial pterygoid muscle C. B.B. 744. E. Fusion temperature of impression compound should occur: A. C. 746.741. E. Below mouth temperature Above mouth temperature As of the skin temperature At the room temperature None of the above 742. D. B. B. D. C. are: A. D. E. 743. C. C. B. D. B. 6% 10% 2% 20% None of the above The disadvantage of heating the impression compound in a water bath is: A. B. C. It may become brittle It may become grainy Lower moles with constituents are leached out The plasticity of the compound may be altered All of the above Generally there is ???? zinc oxide eugenol impression pastes between flow Working time Accelerator Setting time Composition None of the above Dental impression material are hydrocolloids of: A. The flow of the following percentage is allowable for impression compound (type I) at the oral temp of 37º A. B. C. The emulsoid type The suspension type The sol type The get type None of the above Elastomers are:** A. E. E. D. Hydrophilic Hydrophobic Water-loving impression material Potassium alginates . D. 745. The effect of temperature rise above 100ºC on heat cured denture base acrylic resins is: A. Improperly prepared amalgam Improper cavity preparation Perio involvement Particles of amalgam None of the above Reduced occlusal area means: A. E. D. The more matrixes alloys . Produces porosity on the internal portion of the resin. The polysulfide rubber impression material are: A. B. C. C. B. Produces porosity on the external portion of the resin. The weaker the restoration is C. Not sensitive to temperature when curing Quite sensitive to temperature when curing Less sensitive to temperature than silicone rubber The same sensitivity to temperature as silicone rubber None of the above The elastic properties of rubber impression material: A. Prevents porosity on the interior of the resin More fracture potential for amalgam Less fracture potential for amalgam Pulpal involvement Perio involvement None of the above The less mercury remaining in condensed amalgam: A. B. 748. 750. D. C. E. B.E. Improves with time Deteriorates with time Deteriorates when exposed to temperature Improves when exposed to temperature None of the above 749. 751. None of the above 747. B. E. E. The stronger the restoration which contains fewer matrixes alloys and fewer voids B. D. 752. Produces porosity on the surface of the resin. C. D. D. C. The principle cause of failure of amalgam restoration is: A. Is not safe and simple method of bonding D. B. rough part It is free from contamination and roughness None of the above Acid conditioning of enamel and eroded dentine: A. D. conservative clinical approach to the bonding of restorative material B. B. 754. C. Material must be used to clean the surface of the tooth prior to etching The effectiveness of the itchant The chemical and physical nature of the tooth The area and surface of the enamel to be itched Creep in amalgam is the greatest in: . B. None of the above 757.D. 758. Is traumatic approach to bonding materials C. 756. Copper is available for a secondary reaction Copper is not available for a secondary reaction Copper is burnished Copper is fractured None of the above Concerning condensation of restorative gold: A. High copper amalgams are superior if: A. The more voids E. D. C. D. Bonding fails to produce a highly significant retention and good marginal integrity and clinical durability E. It is a perfect substance for bonding It does not conform to the bonding requirements It is the most inorganic. D. E. The effectiveness of the acid etch is dependent on which of the following factors: A. E. C. C. Provides an none traumatic. It may vary widely vary widely and has no influence on the final restoration The degassing procedure is not important It is the Achilles heel of direct gold restoration Clinical tech are more important than the physical properties of restorative gold E. None of the above 753. All of the above 755. B. In regards to the enamel surface: A. Oral administration of short acting barbiturates Intra muscular administration of morphine sulphate Subcutaneous administration of epinephrine Sublingual administration of glyceryl bi-nitrate Putting the patient in upright position Proximal caries on x rays appear: A. Periapical granuloma 765. Angina pectoris C. on a second x ray the radiolucent are moves. Larger than clinically seen C. E. You notice radiolucent area close to the apex of central incisor. C. 760. D. Smaller than clinically seen B. Von Recklinghausen B. Crosses the placental barrier It deposits rapidly in bone It is excreted rapidly by kidneys It is bacteriostatic It produces extrinsic tooth stain 761. Paget’s C. Perpendicular to the outer surface of the tooth Parallel to the outer surface of the tooth Parallel to enamel contour Parallel to enamel-dentine contour All of the following are properties of fluoride except: A. B. C. C. D. E. The surface of enamel rods prisms in permanent teeth is: A.A. Prompt relief can be anticipated in MOST instances from: A. 763. The same 764. The lamina dura is absent in which condition: A. Hypertension B. A patient with history of angina suffers an attack while in the dental chair. Myocardial infection 762. Low copper lathe cut alloy 759. D. B. it is likely to be: . A patient indicates that he takes methyldopa (Aldomet) he is being probably treated for: A. B. 769. All deciduous D. Widening of perio membrane can be seen in: A. Maxillary sinus Incisive foramen Zygomatic process Wall of maxillary sinus At the end of four years. Conservative treatment with sequestrectomy 767. C. C. All deciduous and first permanent molars B. B. D. Cyst Abscess Granuloma Incisive foramen Treatment of Osteoradionecrosis is: A. B. D. Conservative treatment including antibiotic coverage and resection of jaw segment. All permanent except of 3rd molars C. The diagnosis of ortho cases is by: . A periapical x ray of 11 and 12 region shows the vimen. 766. E. D. B. The other feature that can be seen is: A. D. the x rays reveal calcification of: A. C. Scleroderma 768. All permanent 772. floor of the nasal fossa and the median palatine suture. Antibiotic coverage B. 771. C. Lamina dura is actually: A.A. B. Cortical bone Spongy bone Immature bone Cribriform plate perforated by nutrient carnally Bitewing x rays are taken to assist in the detection of caries :** A. Osteosarcoma B. Occlusally Lingually Buccally Gingivally Interproximally 770. C. Incorporate mesh work in wax pattern B. D. E. Preserve vitality of coronal pulp Preserve vitality of entire pulp Preserve vitality of radicular pulp Regenerate a degenerated and necrotic pulp None of the above The objective of pulpotomy is to: . Reversible hydrocolloids impression material in comparison to alginate are: A. Better for undercuts areas 777. 781. Full mouth x ray survey at birth reveals: A.A. When a probe penetrate between tooth and amalgam: A. B. B. C. D. You can increase the retention of Maryland bridge by: A. 773. Benzyl peroxide 780. not always an indication of caries 776. Should be thin and dry 775. Maryland bridges are made of: A. C. C. Measurement of cranium size Recording profile The relation of dentition and the jaw to the cranium Determination of overbite size Determination of jaw size Ten teeth are present Twenty teeth are present Twenty four teeth are present Twelve teeth are present When adhesive is used with polysulphide impression material: A. E. Perforation techniques in the metal cast 778. B. D. The objective of pulp capping is to: A. Initiation of curing process in self cure acrylic resin is achieved by:** A. 774. Nickel chrome 779. Where performing an endodontic treatment on existing root canal filling may lead to fracture of the root B. 783. C. D. E. When there is danger of involving other structures D. What contra indicate pulp capping: A. Preserve vitality of coronal pulp Preserve vitality of entire pulp Preserve vitality of radicular pulp Regenerate a degenerated and necrotic pulp None of the above Absolutely necessary for successful endodontic treatment Not always necessary for successful endodontic treatment Not questioned today as a dogmatic requirement in endodontics Unquestioningly it adhered for successful endodontic treatment None of the above What indicates for a periapical surgery: A. When root canal treatment is faulty C. 782. C. B. B. E. E. E. Cannot be discovered prior to endo treatment May be discovered prior to endo treatment Is most commonly found in maxillary teeth Is most commonly found in mandibular teeth None of the above . None of the above 785. B. E. Accidental exposure on vital young molars When inflammation of radicular pulp is already present When roots are greatly curved and tortuous When anterior tooth is vital and immature with wide open apices None of the above Tow successive negative cultures are: A. B. D. B. Continues after successful endo treatment Stops in most cases following successful endodontic treatment Continues only in mandibular incisors after successful endo treatment Stops in maxillary lateral incisors after successful endodontic treatment None of the above The concomitant perio-periapical lesion as the cause of endodontic failure: A. When the bony defect is so extensive that the edges of the incisors will collapse E. In regards to external resorption:** A.A. D. C. 786. C. C. D. 784. D. size and direction of roots and root canals C. Remove noxious material since it may be forced to the apical foramen resulting in periapical infection C. Outline form B. The length of the tooth is established by: A. B. Medium alloy “Type II” Hard alloy “Type III” Extra Hard alloy “Type IV” All of the above 792. Determine the number. None of the above 789. Internal external relationship D. C. None of the above 790. 791. D. should be undertaken at frequent intervals during instrumentation to:** A. All of the above 788. location. Irrigation in root canal treatment. Stop instruments from going beyond the apical foramen E. The age and shape of pulp chamber. B. Aid in the diagnosis of periapical hard tissue lesion B. shape. Confirm the length of root canals D. D. Intra-coronal preparation E. Destroy all micro organism in the canal D.787. Evaluate the adequacy of the complete root canal filling E. To achieve optimum cavity preparation which of the following factors of internal anatomy must be considered: A. X rays are used in endodontic treatment to: A. C. C. Which of the following varieties should be made in the proximal occlusal cavity preparation in deciduous teeth compared to permanent ones: . Good undistorted pre-operative x ray Adequate coronal access to all canals Adjustable endo millimetre ruler Definite repeatable plane of reference to anatomical landmark on tooth All of the above Which of the following Gold casting alloys are available: A. in addition to the direction of individual root canals. Removes cementum falling from the canal B. E. B. E. Refer child to pedodontist B. B. Teeth adjacent to extracted teeth Teeth on both arches on same side The remaining teeth in the mouth Teeth directly opposite to the extracted tooth Teeth on the same quadrant 794. Gradual transformation of the reduced enamel epithelium 796. D.???? A. 793. Schedule child for General Anaesthetic session 798. B.A. E. B. The occlusal isthmus should be proportionally wider The occlusal lingual walls need not to be extended to self cleansing areas It is not necessary to include fissures in the occlusal outline The lingual angle should be sharper The axio pulpal line angle should not be bevelled 10 years old boy looses permanent mandibular molar. Loss of arch length Loss of speech sound Loss of facial contour Loss of vertical height Loss of free way space 795. X rays C. The most common reason to refer a child to a pedodontist is problem with: . D. Classifications B. Which of the following approaches would seem to be your alternative: A. D. Essential for the diagnosis and treatment plan of orthodontics is: A. you decide at an initial appointment that you can not handle a child due to lack of co-operation. C. what is affected: A. D. E. Plaster models 797. Down growth of oral epithelium which replaces the reduced enamel epithelium Proliferation of inner enamel epithelium Proliferation of outer enamel epithelium Down growth of oral epithelium which undermines the reduced enamel epithelium E. C. the reduced enamel epithelium merges with the oral epithelium and consequently…. C. During teeth eruption. C. As a general practitioner. Send child home until he/she has to co-operate C. MOST common consequence arising from premature extraction of deciduous molars is: A. Length 804. Primary enamel cuticle Secondary enamel cuticle Acquired enamel cuticle Cementum Oral epithelium fused with reduced epithelium to form: A. B. Change in the interocclusal dimension 801. A very quick and wide separation of teeth causes: A. D. 800. B. 8 years child has a badly broken deciduous molar what is the best material to restore it: A. TMJ dysfunction B. 799.A. D. D. B. Mesio version B. Amalgam Gold Composite GIC When tooth is twisted along its long axis. Width C. C. C. The final deposition of ameloblast occurs:** A. can get: A. B. Changes in the vertical dimension C. Junctional enamel epithelium 803. C. Rampant caries Behaviour management Endodontic treatments in primary teeth Space maintainers Gingival inflammation Vasodilation Wider spaces Necrosis of bone A patient who has lost several teeth in an otherwise healthy mouth. Depth B. 802. 805. it is called: A. The increase of mandible about year 5 and 6 is mainly at: A. Disto version . C. D. A tooth is expected to erupt when root development is: A. The function of varnish: A. Back pressure porosity 815.C. Ectopically erupting permanent molars 812. The outcome of rapid wax burn out is: A. Crowding of anterior permanent teeth is directly affected by: A. A full x rays is recommended in children by age of:** A. To close midline diastema B. To help eliminating excess mercury B. Lingo version D. To reduce initial marginal leakage “Short-term leakage” B. Hawley appliances are used:** A. Cracking of the investment B. To condense margins . 2 years for uncooperative kids C. 2 years. To prevent long term leakage 813. The percentage of malocclusion after early loss of deciduous teeth is: A. Turner’s tooth is:** A. The MAIN purpose of burnishing is: A.first visit B. Due to infection of primary tooth 814. 60% 808. 3-5 years 807. ¾ of its full development 809. Maintain the normal relationship of the adjacent teeth until the canine erupts 811. Torsion 806. Helical spring is used to: A. Premature loss of deciduous molars 810. Hybrid composite 823. The function of face bow is: A. What happen to etched composite after settings: A. Mesio lingual cusp 824. D. The MOST common cause for midline fracture is: A. Expand Contract Contract and expand Expand and contract Which of the following muscles elevates the lower lip: A. Fatigue 819. Polishing of filling 816. D. The palatal canal of maxillary molars is found Under: A. B. C. Impact B. Orient maxilla to TMJ 820. 817. C. Hypoplasia as seen in x rays:** A. 822. E. F. Thick enamel surface Thin enamel surface Sometimes large pulp chamber Can not be detected on X rays Substitute for the missing wall so adequate condensation forces can be applied Permit re-establishment of proper contact lines Restrict extrusion of amalgam and prevent formation of an “overhang” Provide adequate physiological contour for the proximal surface Provide an acceptable surface texture to the proximal surface All of the above Which composite is used in load bearing areas:** A. Orbicularis oris 818. Obturator in cleft palate plate is maintained by: .C. C. Function of matrix band: A. B. Disto lingual cusp B. D. 821. B. and distribute metallic oxides as they are formed 830. What do expect after successful pulpectomy in the periapical area: A. B. In compression B. D. Drug interaction with patient’s medicines Injecting into vein Hypersensitivity Toxicity . Porcelain bonded to metal is strongest: A. B. What is TRUE: A.5mm 829. Apical foramen is closed by cementum calcified tissues 827. C. Boiling point of acrylic < boiling point of water 831. 0. To protect alloy from oxidation. Mesial to posterior retainers 826. 1. A fold of mucous membrane 828. Distal to anterior retainers B. In tension 832. C. 1mm B. D. What is the MOST adverse reaction to lignocaine: A. Boiling point of acrylic > boiling point of water B.5mm C. Boiling point of acrylic is similar to that of water C. Cohesion Atmospheric pressure Retention in the defect Patient support it with the tongue 825. What is the function of flux: A. What is the minimal labial reduction for porcelain metal crowns: A. Frenum is consisted of what kind of tissues: A.A. In a fixed moveable bridge where should the moveable connectors “non rigid” be placed:??? A. 833. A 29 year old lady presents with mandibular second molar associated with radiolucency of 1 cm diameter and paraesthesia of mental nerve. There is no other symptoms: A. B. C. D. 834. Extraction and curettage Root canal treatment and antibiotics Blood test, extraction and biopsy Extract and pack with white head’s varnish Periodontitis is a disease that has: A. B. C. D. 835. A slow progression Rapid progression Cyclic or burst progression “active and inactive phases” Intermittent progress In regards to apically displaced flap; which is TRUE: A. B. C. D. 836. Does not preserve attached gingivae Does not lengthen crown of tooth Is a pocket elimination procedure A&C Which is NOT TRUE about occlusal trauma: A. B. C. D. E. 837. Cemental tears Bone loss Mobility True pocket formation Bleeding in periodontal ligament Which is the MOST significant clinical feature of periodontal disease:** A. Bleeding B. True pocket formation and apical migration of attached gingiva 838. Hypodontia can be seen in: A. B. C. D. 839. Cleidocranial dysplasia “dysostosis” Down’s syndrome Papillon le fever syndrome Rickets Hyperdontia can be seen in: A. Down’s syndrome B. Cleidocranial dysplasia “dysostosis” 840. Which of the following does not carry a risk of infection from hepatitis B patient: A. HBs Ag antigens B. HBs Ag C. HBe Ag 841. Which is the MOST conservative treatment for periodontal disease: A. Oral hygiene, sub-gingival debridement, regular review and maintenance B. Surgery, sub-gingival debridement, regular review and maintenance C. Oral hygiene, sub-gingival debridement 842. Filter is used in x ray machine to: A. Reduce exposure time B. Removes low energy x rays 843. A patient 8 years old has 3 of first premolars erupted with swelling on the ridge of the un-erupted premolar. X ray shows a fully developed crown and ¾ roots development with no other pathology. What is your management: A. Remove the dentigerous cyst B. Soft tissues recision to allow eruption C. Soft tissues recision accompanied with orthodontic appliance to help with eruption 844. Loss of the first deciduous molar in 10 years old child required: A. Band and loop to maintain space B. Evaluate the case radiographically and then decide whether space maintainer is needed or not C. No treatment 845. Palatal root displaced into the antrum while extracting; what is your decision to retrieve it: A. Through the alveolar B. Surgical opening of canine fossa C. Nasal antrostomy 846. Which one of the following is expansile lesion of jaw bone:** A. Odontogenic keratocyte B. Central haemangioma C. Radicular cyst D. Osteomyelitis 847. The MOST frequent retained deciduous teeth in permanent dentition are: A. B. C. D. E. 848. Upper lateral incisors Upper central incisors Lower central incisors Second lower molars Second upper molars The MOST frequently synthesized substance by Streptococcus mutans is: A. B. C. D. Liven Fructose Glucan Glycogen 849. Benzodiazepine and diazepam in 5-10mg oral dose used for oral sedation in dentistry DOES NOT give: A. B. C. D. 850. A good analgesic effect if given 1 hour prior to dental sessions Would be reversed by flumazepil because it is a Benzodiazepam Post operative headache There is a profound amnesic action and no side affects Formcresol fixation is used in deciduous dentition in: A. B. C. D. Necrotic pulp Carious exposure Mechanical exposure Periapical disease 851. After you have successfully treated an Angle’s Class II division I malocclusion. The ideal Class I incisor relationship has been produced and 14, 24 were extracted. The arches are now well aligned. What molar occlusion will there be at the end of treatment when all spaces are closed: A. B. C. D. E. 852. Full unit Class II ½ unit class II Class I ½ unit Class III Full unit Class III The tensile nodes are located at: A. The mandible angle B. The jugular-digastric interaction C. Mylohyoideus intersection D. D. D. C. B. B. Reduces the flexibility of films B. C. C. B. Sulphur oxides Oxygen Chlorides Over trituration . Metallic Plato backing the intra oral films are for:** A. 100-120 nm 200-300 nm 400-430 nm 470 nm or 450-500 nm When is LEAST required gingival groove: A. C. Microscopy C.D. D. C. Herpangina is the MOST reliable diagnosis is by:** A. 857. D. Internal carotid level 853. 858. B. When restoring with GIC for abrasion When restoring with GIC for root caries When restoring with GIC base and composite lamination When restoring with amalgam Corrosion and discolouration of amalgam restorations is usually caused by: A. 856. 859. Reduces patient exposure to x rays C. 5 minutes at 20ºC At least 10 minutes Until it clears up 2 minutes at 40ºC The developing time for dental x ray should be: A. B. The fixing time for dental x ray should be: A. Immunofluorescence B. Serology 854. 5 minutes at 20ºC At least 10 minutes Until it clears up 2 minutes at 40ºC What is the range of the visible light cure beam: A. Increases the bending capacity of films 855. Patient presents with fever of 39ºC. C. Use an inverted cone bur Use a flat fissure bur Parallel to occlusal plane At right angle to the long axis of tooth None of the above 863. When you find ditching in an amalgam filing you would: A. Repair defect with unfilled resins 861. Antibiotic. 866. Complete debridement of root canal. D. 865. pain. Why do we itch enamel for composite restorations: A.860. Radiograph shows an enlargement of periodontal ligament space of 11 which has a large restoration without a base. C. analgesic and antibiotic D. What would your treatment be: A. analgesic followed by root canal treatment after remission of acute phase. apply a sedative dressing with corticosteroids 864. B. B. What is the reason that pulp calcified after trauma: A. Which is TRUE in regards to the preparation of occlusal rests: A. E. C. Remove restoration. Cervical finish line of full veneer crown preparation should be placed: A. Mucous membrane carefully separated from periosteum C. Base is wider than free margin B. Base has an adequate blood supply . B. The intensity of the blow was too low to cause pulp death 862. D. swelling of upper lip and nose. Recision and antibiotic B. Replace the defective filing B. C. D. E. Just supragingival whenever is possible According to the depth of gingival crevice Subgingival to reduce ability of recurrent caries At the junction of tooth and amalgam core To increase surface area To decrease surface area Does not really change the surface area Increase the chemical bonding capability Decrease the chemical bonding capability All of the following are requirements of mucoperiosteal flap except of: A. B. To ensure excess mercury reaches the surface 872. B. 868. C. B. D.D. 873. Aesthetic Phonetics Gothic arch tracing Swallowing Zinc oxide impression material: A. C. Hedstrom file C. Too wide Too short To narrow To long Which of the following is more prone to crack: A. Flap wider than bony defect that will be present at conclusion of operation E. Is a thermoplastic material 869. A lateral incisor labial to the arch needs to be restored in normal alignment with PFM retraction. D.5% hypochlorite sodium B. C. D. B. EDTA 871. How will the tooth appear: A. Current intensity is too high Current intensity is too low Dispersion plate not applied to patient None of the above How do remove the smear layer in root canal treatment: A. 870. Why do you over pack amalgam: A. C. Which of the following will NOT be used in determination of vertical dimension: A. The adhering of tissues on the surgical electrode usually means: A. May cause irritation to mucosa B. Buccal of lower molars Lingual of lower molars Lingual of upper molars Buccal of upper molars . Mucoperiosteum is carefully separated from bone 867. Use of 0. D. Flexibility of the retentive clasp arm depends on: A. Soluble in chloroform B. Too weak for narrow canals 880. B. To provide guide planes 878. B. To orient cast to surveyor C. To orient cast to articulator B. Length Cross section Material Degree of taper All of the above In vital pulp therapy. 877. Proximal cavosurface walls in Class II preparation for the reception of an amalgam. An irregular shaped void on surface of a gold cast would indicate that: A. D. D. 879.874. Acute angle Right angle Obtuse angle 45 angle Why are three tripod marked on a cast being surveyed: A. Reduces new caries and hamper the progress of existing caries . D. Reduces new caries and hamper the progress of freshly established caries B. B. Should be finished at which angle to external surface: A.5mm 876. C. 4-5mm Approximately 2mm Less than 2mm 1-1. C. E. what is the optimum depth for a pin hole in a tooth: A. C. B. A fragment of investment had been carried into the mould Air carried into mould Burning out of wax was inadequate The powder/water ratio for the investment was too high The MAJOR disadvantage of Gutta Percha is: A. 875. D. C. What effect do fissure sealants have on caries progression: A. In regards to connectors on dentures. Secondary caries may develop 886. bottom of the fissure B. What is CORRECT in regards to periodontal surface area in mandibular teeth: A. Condyle C. which of the following is correct: A. Relieve cast from the inside Take a new impression and make new crown Burnish margins Use thick mix of cement In regards to marginal leakage in amalgam: A. walls of the fissure 887. First molar> first premolar> second premolar B. Canine> lateral incisor> central incisor 884. Coronoid process B. D. E. Canine> first premolar> second premolar C. Seal the margin with fissure sealant would prevent further breakdown C.3mm what would you do: A. Major connector should be rigid as possible B. Cast crown fits on die but not on tooth. What interferes with maxillary denture in posterior vestibular fold: A. B. Adjacent tooth and contralateral teeth Contralateral and opposing teeth Opposing and adjacent teeth Test only suspected tooth/teeth All of the above 883. discrepancy is about 0. D. What control tooth or teeth should be used when testing a suspected pulpally involved tooth: A. Masseter muscle .881. Minor connector should engage undercuts 882. 885. Pit and fissure caries start at: A. C. The wider the gap the better the chance of secondary caries B. B. C. What is the Bilaminar Zone: A. C. The zone where buccal and lingual forces are balanced 894. The zone where displacing forces are neutral B. patient has Class II Division II type with deep bite. Formed of. C. or thin plates.888. Which of the following is TRUE: A. Plaque accumulation 891. two laminae. When lateral incisor is lost. Contraction of lateral pterygoid Contraction of temporalis Contraction of the suprahyoid muscles Contraction of the infrahyoid muscles Relaxation of all muscles so that the only forces on the mandible are the forces . B. Which of the following is contra indicated: A. Which is the neutral zone: A. 12 hrs Immediately after fabricating it After been left in water for an hour Wait for an hour before pouring The MAIN CAUSE of gingivitis in partial dentures patients is: A. Non-rigid connector with central incisor as abutment 893. or having. B. how long should have been made prior taking impression: A. Brightness B. Mesial drift causes unseating of the distally placed connector 892. Which of the following DOES NOT cause depression of the mandible: A. E. Should be placed on the longer retainer B. Acrylic self-cure special trays. Value 889. D. Saturation of hue C. Movable component of the non-rigid connector in a fixed bridge is placed. Which is the distal attachment of superior hard lateral plate 895. In regards to shade. Chroma is: A. Placement of dentures B. D. 890. Fixed bridge with canine and central incisor as abutment B. Your management is: A. Antibiotic coverage C.M. Surgical incision B.5ml of 1:1000 adrenaline with oxygen administration 903. I. 2mm to achieve a good resistance form C. Which of the following procedures will not achieve sterilization: A. Which of the following is the MOST appropriate related to hardness: A. How long it would take to notice significant reduction in radiolucency after finishing a root filing for a tooth with a periapical lesion: A. 6 months B. Boiling water at 100ºC “210ºF” for 2 hours C. Tongue thrust 900. To increase the setting time of phosphate cements you would: A. When treating a tooth with a non-vital pulp with a fistula presented. Hot air at 160ºC “320ºF” for 90mins B. Use a cold glass slab 902. Porcelain>Enamel>Tungsten carbide>amalgam>acrylic C. Tungsten carbide>Porcelain>Human enamel>acrylic B. The usual root canal procedures for non-vital teeth and no special procedures for fistula 901. Class II Division I B. 27 years old female. 1mm 898. fistula should be treated by: A. Porcelain>Enamel>Tungsten Carbide>Amalgam>Acrylic 897. Autoclave at 121ºC “250ºF” under 15psi for 20 mins . How much would you reduce a cusp to be replaced with amalgam onlay: A. o. 3 months 899. The major cause of mentalis muscle hyperactivity is: A. shows sudden oedematous rash and collapses after an injection of barbiturates.against the gravity 896. 1 month C. 2 mm to achieve a good retention form B. D. Which of the following pre existing conditions could be responsible for the post operative bleeding: A. 907. Blood pressure reading of 180/110 Gastric ulcer Elevated prothrombin time A & D are correct None of the above Long bone growth by: A. B. C. she has had a rubbery.D. Dry heat at 177ºC “350ºF” for 60mins E. B. All of the above will achieve sterilisation 904. A patient states that for ALMOST a year now. D. C. D. E. 905. firm. C. This MOST likely is: A. Mitosis in osteoblast Mitosis of osteoblast Appositional growth in cartilage epiphysis Interstitial growth in cartilage epiphysis What is TRUE in regards to oral lesions of reticular lichen planus: A. E. Never accompanied with skin lesions Always accompanied with skin lesions Lesions may present anywhere Lesions may present on legs Lesions may present on arms Bone graft method that has shown the greatest osteogenetic potential is: A. 909. Lymphocytic bone graft Freeze-dried bone graft Heltozygo?? Marrow graft Cortical bone graft Cancellous bone graft 908. painless nodule within the substance of parotid gland. B. B. 906. Use lead collimator . 50 years old man presented after a full mouth extraction complaining that he “bled all night”. D. B. E. E. C. Mucocele Lymph node Benign mixed tumour Squamous cell carcinoma Sialolith with encapsulations The best method to radiate a specific area of the head is: A. C. D. D. C. Nasal floor Cranial vault Occlusal plane Naso maxillary complex Anterior cranial base 914. D & E . F. Child with less than normal number of teeth. on x rays it shows with two roots and two roots canals. 7 years child with Class I malocclusion. B. B. D.910. What is your management: A. E. 915. Which is NOT CHARCTERISTIC of dentinogenesis imperfecta: A. E. C. Clinical indications of pathogenic chronic periodontitis: A. Oral screen Head cap therapy Inclined plane on mandibular anterior teeth Hawley plate Expansion screw plate Tongue thrust with tongue to lip swallow is seen in: A. Dentinal tubules are more than usual 912. Mobility Dull pain on closing Presence of true pocket Apical migration of gingival epithelium Presence of subgingival calculus C. E. adequate arch length. C. slight version of maxillary Class I. Dilaceration Gemination Fusion Concrescence Taurodontism The MOST stable area to evaluate the craniofacial growth is: A. C. Density of lamina dura 911. Incompetent lips 916. 913. D. B. B. D. The best reading on radiograph to diagnose ankylosis in deciduous molar is: A. E. your diagnosis is:** A. mandibular lateral incisor is larger than usual. B. To prevent exposure of a ?????? on a permanent root. B. B. E. Dental fluorosis and enamel opacities . C. 918. teeth should be splinted: A. D. In advance periodontitis with marked mobility. E. Free gingival autograft Sub-epithelial tissue graft Apically positioned graft Free gingival graft Modified wide flap Crosses placental barrier Deposits in bone Excretes rapidly by kidney Bacteria static Produces extrinsic tooth stain Two conditions of enamel facilitate post eruptive uptake of fluoride element: A. Double flap Stripping procedure Full thickness flap Apically positioned flap Split thickness flap What is TRUE in regards to primary occlusal trauma: A. D. Which is the procedure of choice to obtain coverage of the root surface: A. B. C. Hyper mineralisation and surface dentine B. the dentist BEST approach for elevating flap is to use: A. 921. Mobility caused by excessive forces on normal periodontal ligament 919.917. Surface demineralisation and hypo mineralisation C. exposed wide area of dental roots. 922. What does contra indicate distal wedge in molars’ area: A. C. Distal fluting Long attached gingiva Sharply ascending ramus that limits space distal to molars Supra bony pockets distal to molars Which of the following is not a property of Fluoride ion: A. 923. D. E. Examination reveals area of gingival recession. D. To improve comfort and function 920. C. Long edentulous span which will lead to damage of abutments 932. 2:3 B. 1:1 931. 6-8 weeks 4 weeks 6-8 months 4 months The OPTIMUM crown to root ratio for abutment tooth is:** A. C. Flexibility of the retentive clasp arm Does not relate to: A. D. Effective in incorporated into dental plaque B.924. Patient has been coming to your clinic for several times complaining about soreness under the denture. Infectious mononuclears 928. In regards to topically applied fluoride : A. Less vibration on patient 929. Prevents distortion when impression is removed out of the mouth 933. what would you do: . E. The FIRST advantage of using 100000 rpm and over rotors is: A. 930. B. Inhibits acid demineralisation of enamel 925. Can not be perforated 927. 926. How long it would take to see the dentinal bridge after direct pulp capping by using Calcium hydroxide:** A. C. Length Cross section Material Degree of taper Under cut area Protrusive movement in wax: A. Elasticity of impression material will lead to ideally: A. D. B. LEAST use of blood count: A. What does contraindicate bridge works: A. Haemangioma 936. How would you treat hyperaemia “hyperaemic tooth”:** A. Patient comes to you complaining of pain in a tooth. 942. B. Purplish lesions on the buccal mucosa that have been there since birth. Calcium hydroxide C. Check occlusion of lower buccal cusps 934. Zinc Oxide and eugenol cement B. Acromegaly Paget’s disease Giant cell lesion Primordial cysts Dental cysts For fissure and sealant treatment to be a part of the primarily retentive care: . what would you do: A. Elevators are not used in: A. In arcon the condylar element is in the lower compartment 935. Bilateral symmetrical swelling of the mandible of a child is likely to be caused by: A. Carious exposure of pulp in tooth that has been painful for weeks 938. Use of inhalation general anaesthesia: A. X ray. the tooth is filled with composite long time ago. Halothane should not be less than 5% B.A. What is the difference between arcon and non arcon articulator: A. What is contraindicated to the use of calcium hydroxide for pulp capping: A. Accidental exposure of pulp B. remove filling and restore with temporarily filling 940. D. Carious exposure of pulp in otherwise asymptomatic tooth C. C. Oxygen must not be less than 30% 941. Dividing third lower molars roots 937. E. the diagnosis is: A. Corticosteroid paste 939. A. When you apply a pressure of 0. Away from edentulous space Adjacent to edentulous space Near fulcrum line Away from fulcrum line 948. D. 946. 4 mm indicates periodontitis 945. 949. CEJ to base of pocket B. Top of the gingiva to the base 944. Missing proximal contacts 947. Which of the following elements is not found in normal periodontal membrane: A. B.25N to measure pocket depth: A. Periodontal pocket is measured between: A. D. C. Enough proximal surface B. Respiratory depression . B. E. Fibroblast Epithelial cells Erythrocytes Vest cells of malaise Inflammatory plasma cells and lymphocytes Which of the following situations make periodontal disease more sever: A. Between crown and cement Between core and cement In the crown and the root In the core and the margin preparation Which is NOT a result of toxic dosage of local anaesthetic: A. where does failure occur: A. D. Place sealant on teeth which are at high risk of caries B. Hypotension C. Too wide bucco lingual embrasure C. C. Place sealant on newly erupted teeth 943. C. Angioneurotic oedema B. The auxiliary occlusal rest on tooth for partial denture should be placed: A. B. A vital tooth has a crown cemented to a pin retained amalgam cored. To relief pulp pressure 955. The main of damaged gingival tissues after placing rubber dam is: . On X ray you found the Gutta Percha cone extending 1mm beyond the apex without any symptoms. Leave as is until any complications occur 954. D. Redox potential favours growth of anaerobic bacteria It is caries immuno-suppressive It is adrenergic Affects neutrophils and chemotactic factors 952. Hypertension 950. Swallowing will aid in the diagnosis of: A. Cross cut fissures at ultra speed 957. What is the main purpose of using corticosteroids in pulpal obturation material: A. D. For their antibiotic action B. Branchial cyst Thyroglossal duct cyst Ranula Retention cyst Globulomaxillary cyst What is not true about tobacco smoking: A. What nerve supplies upper first molars: A. Apiectomy C. Posterior and mid superior alveolar nerve: 956. what would you do: A. Leave as is until any complications occur 953. Remove restoration material until you are able to withdraw the Gutta Percha cone B. C. Apiectomy C. what would you do: A. B. Remove restoration material and retreat B.D. C. On X ray you found the cement of previous root canal treatment is extending 1mm beyond the apex without any symptoms. 951. B. The roughest surface on cut tooth structure: A. E. For their antiinflammatory action C. 958. Anywhere beyond the survey line C. What is characteristic of fibrotic gingivitis: A. Cover the occlusal and buccal cuspid by porcelain B. You have patient with Class II division 2. What is the main cause of bilateral cheilosis: . It acts as indirect retention 959. Because of the high casting shrinkage of porcelain 964. As much under the undercut as possible B. D. Can only be treated surgically 962. Haversian canal around bony canals B. A predetermined amount of undercut 961. Irregularly arrayed tabullae?? 963. which of the following is contraindicated: A. The distance between holes is big The distance between holes is small The punctured holes are too big in size Not using lubricant when placing rubber dams The advantage of using the lingual plate on lingual bar is: A. Is phenytoin induced gingivitis and only seen on intra lateral papilla B. Why we do not use porcelain in long span bridge works: A. B. Retention in precision attachment is achieved by: A. How will cover buccal bicuspid for lower premolar when making a metallic porcelain crown: A. Maryland bridge 965. How much under cut area a clasp arm should engage:** A. C. Bone is characterised by:** A.A. Cover just buccal cuspid by porcelain 966. Frictional resistance 960. Cantilever bridge B. Vitamin B deficiency 967. The MOST common sites for squamous carcinoma in the oral cavity are:** . 10 to 15% 973. When the neck of the condyle is fractured. C. What is your management: A. carefully suture and give Amoxicillin or Vancomycin C. Short vertical dimension B. requires surgery. with INR value of 3. Pigmented naevus can undergo malignant: A. B. what is the your management: A. Stop warfarin. Always B. Chronic oral antral fistula for some time after the extraction of maxillary first molar. Wash the antrum 972. D. Stop warfarin. what muscles determine the movement of the superior segment: A. Single phase materials Multi phase materials Extra hard The same as bonding martial 969. What sort of alloys do you use for bridges: A.0. Give Amoxicillin or Vancomycin and suture carefully B. Anti-biotic and nasal decongestant C. C. What sort of material do you use for the fabrication of Maryland bridges: A.A. Lateral pterygoid Medial pterygoid Temporalis Mylohyoid 970. Never C. Hard C. Patient with prosthetic heart valves. High sensitivity 968. B. Ductile B. Surgical closure B. start heparin. carefully suture and give Amoxicillin or Vancomycin 971. D. Palate and gingivae B. B.A. D. A localised purulent infection of the skin Diffuse purulent infection of the skin Staphylococcal osteomyelitis Impetigo What is TRUE in regards to Basal Cell Carcinoma . Smaller focal spot C. E. 978. A patient has painful lesions on her buccal mucosa. Tongue and palate 974. Biopsy report shows acantholysis and supra basilare. Increase which of the following will decrease density of radiograph: A. your diagnosis is: A. Larger focal spot B. C. B. Tongue and floor of the mouth C. The MOST common staphylococcal infections is: A. Milliampere Time KvP Kilovoltage Object-film distance Focal spot-object distance Which of the following will increase sharpness: A. what is TRUE: A. 976. C. D. May be associated with deafness C. 980. D. B. C. Sex linked disorder of bones that develop in cartilage 977. 975. D. Commonly seen in ethnic groups Commonly an amalgam tattoo Commonly oral melanoma Commonly melanotic naevus What I TRUE in regards to osteogenesis imperfecta: A. Increase object-film distance 979. Manifests with blue sclera B. Pemphigus vulgaris Bulla lichen planus Erythema multiform Systemic lupus erythematosus Oral mucosal pigmentation. C. B. R. A pocket of more than 4 mm depth B. 6 5. probe: A. E.A. D. Prevent apical migration of junctional epithelium 984. Allow the growth of connective tissue in contact with surface C. Metastases is common Erodes bone More common in oriental races Cannot occur in oral mucosa according to definition In severe periodontitis. Amalgam . 982. Only 1mm of attached gingiva remains C. D. Patient is very young and can not understand direction Patient is physically handicapped and unable to hold the film Film should never be held by the dentist There is a lack of time and radiograph is essential Common cause of poor diagnosis in avulsion replantation: A. Posterior vital molar with core the best material to restore it is: A. B. Get stopped by calculus Goes beyond connective tissues of junctional epithelium Touches coronal end of junctional epithelium Touches the middle of junctional epithelium Touches sulculuar epithelium Characteristic of mucogingival involvement: A. C. External resorptive defects 987. Prevents apical migration of junctional epithelium B. C. C. D. C. The critical plaque PH is: A. The role of Guided Tissue Regeneration G. B.5 985. 986. When it is acceptable for patient to hold radiographic film packet in the patient’s mouth: A.5 4 4.T. B. B. Pocket extends to the mucogingival junction 983. is: A. D. 981. Cervical third 994. In regards to denture stomatitis: A. MOST LIKELY it is. D. Aesthetic 992. More dentinal tubules. Area of radiolucency surrounding the apex of tooth 995. 998. The MOST unfavourable root fracture: A. The GREATEST reliable finding to confirm a necrotic pulp is: A. C. you notice a hard dark brown spot on the adjacent tooth just below the contact point. When preparing Class II cavity. more intertubular and less peritubular More peritubular. B. Demineralised enamel 996. Which of the following is important consideration when deciding whether to design an upper partial denture without anterior flang: A. intertubular and peritubular Pulp with multiple microabscesses will cause eventually: . Zinc Oxide and eugenol impression paste: A. some intertubular and peritubular Some dentinal tubules. Due to over growth of some normal commensal of oral cavity 993. in TMJ area: A. The function of incisor pin of an articulator: A. In cavity preparation 1mm below DEJ what is seen: A. A. Can not be used in areas with undercuts 991. When restoring with composite resins. some intertubular and dentinal tubular Equal amount of dentinal tubules.988. why do we do the cavo surface bevelling: A. The amount of labial alveolar bone resorption 990. Horizontal and vertical overlap 989. When opening the mouth. Initial rotation followed by translation of condyle 997. Loss of blood supply B. B. Needs little maintenance B. To prevent cervical resorption defects following bleaching: A. Well constructed complete denture: A. Endodontic therapy completed on tooth with periapical radiolucency. Marked reduction in size of radiolucency is expected in approximately: A. Immediate extraction of any poor teeth under local anaesthetic with antibiotic coverage B. Segmental dental clearance and closure to eliminate problems C. 58 years old male has had a 60 yo WM course of radiation given for carcinoma of tongue. Clearance of poor dentition followed by hyperbaric oxygen treatment plus a primary closure of wounds under antibiotic coverage . Patient complains of pain associated with poor dentition. Loss of coronal tissues 1005. 1003. Necrosis 999. D. D. Which muscle acts on the disto lingual contour of lower denture: A. No dental treatment may be due to neuronic of neoplasms D. C. C. The MOST common curvature of palatal root of maxillary first molar is: A. Remove Gutta Percha at least 2mm below CEJ or above the crest of alveolar bone and isolate 1002. 1004. One year 1000. B.A. The dental management would be: A. Less than a week for adjustment and total success C. Mentalis Masseter Mylohyoid Buccinator Distal Mesial Buccal Palatal The reason that endodontically treated teeth are weak is: A. Adverse effects and decrease taste sensations 1001. Pulpalgia Internal resorption Hypercalcification within root canals All of the above The MOST common occurrence after direct pulp capping is: A. Formation of dentine . D. Occasional sensitivity in a shallow class I amalgam restoration after two days would be managed by: A. why do teeth maintain contact: A. Replace the composite B. On examination of composite restoration you find a dark attain: A. E. Replace old filing immediately Oxide Zinc and eugenol Using thicker mix of cements Tell patient the discomfort will disappear after 4 t o6 weeks Ledermix Pulp capping in mature tooth may be followed by:** A. Between bisque stage and glazing stage Between preheat and opaque stages Between opaque and bisque stages Between one opaque and two opaque stages Too thick application of pure gold surface conditioner Contamination at the porcelain metal interface Under firing the opaque layer All of the above Attrition in elderly. “Pop off” of a porcelain veneer from under the lying gold crown is due to:** A. When should not contaminate metallic framework during fabrication of porcelain fused to metal crown: A. C. 1011. 1008.E. Repair with unfilled resin C. 1012. No extraction as radionecrosis is an important sequelae 1006. C. D. D. B. B. B. Increased interocclusal distance C. D. Signs of reversible pulpitis 1010. 1009. C. Building bone around the fundus of alveolar bone and deposition of cementum B. B. Apply topical fluoride at the margin 1007. C. B. Regeneration of cementum B. Smooth surface C. Long junctional epithelium 1019. Good oral hygiene and fluoridation is LEAST useful in preventing caries of: A. Pit and fissure B. Initial condylar guidance of 25 degree was wrong is changed to 45 degree. Hetero formation is better at low sugar concentration C. Mobility 1020. D. Why is the frequency of carbohydrates intake more important quantity: A. 1017. B. Regeneration periodontal surgery: A. Low number of streptococcus mutans B. What is NOT TRUE about gingivitis: A. Spoon shape with rounded margin 1018. Extirpate the pulp that is obviously inflamed Place ZOE dressing to sedate the pulp Ask patient to come back in six months Repeat restoration What is the shape of occlusal rest: A. The growing root Bone growth Vascular pressure The developing periodontal ligament 1014. Reduce cusps height C. What changes will you make to achieve balanced occlusion: A. D. C. Patient complains of sensitivity. Decrease incisal guidance B.1013. Homo formation is better at high sugar intake . C. Increase compensate curve 1015. what is your next step: A. Inaccessible area 1016. on examination you found a composite restoring a good cavity preparation without any secondary caries. The MOST likely factor contributes to tooth eruption is:** A. Gingivitis is not caused by: A. B. 1026. Primary lesion is not contagious B. Viral infection 1022. Both the primary chancre and the secondary mucous patch stages of the disease are highly infectious C. B. Ulcers. All of the above . 2-5 hours 5-12 hours 12-30 hours 30-48 hours 48-96 hours Myxoedema occurs due to:** A. The elimination half life of Diazepam is in the range of: A. The spirochetes disseminate rapidly throughout the body within 24hour after contact B. 1023. Diabetes B. D. Hypersecretion of the thyroid Hypersecretion of the adrenal Hyposecretion of thyroid-hypothyroidism Hyposecretion of the adrenal 1024. reduced rete pegs will be diagnosed as: A. D. Spirochetes disseminate in 24 hours 1027. D. C. necrosis and plasma cells at the basal membrane with atrophic thin areas. Only the lesions of the primary and secondary stages are contagious D. Oral lesions are not seen in less than 1% C.D. Which of the following is seen in benign mucosal membrane pemphigoid:** A. C. C. Tzanck cells Intraepithelial vesicles Histopathology like aphthous ulcer Scarring of the conjunctiva In syphilis: A. B. E. Restricted diffusion of acid through plaque 1021. Desquamative gingivitis 1025. Which of the following is TRUE about syphilis: A. C. C. Review in yearly intervals 1031. B. Are predominantly anaerobic Must be treated by antibiotics Must always treated by surgery Change fro aerobic into anaerobic 1033. D.1028. what is the first Maintain airway and place in supine position Give insulin injection Administer oxygen Inject adrenaline 1034. Perform mixed dentition analysis B. C. E. moist skin and dyspnoea. A. B. D. what is TRUE: A. C. Apply a fixed appliances E. What is your management: . Ask the patient to come after the deciduous teeth fall off and complete permanent dentition erupts D. Patient with weak pulse. Single retroclined upper incisor in 9 years old. Staphylococcus aureus can cause which of the following infection: A. Thyroiditis Pancreatitis Osteomyelitis Scarlatina Pneumonia 1030. In regards to paracetamol: A. D. B. thing to do: A. Which of the following is not true about warfarin. What your next step would be: A. A 10 year old child presents with crowding of the dentition and desires correction. D. INR of 3 is enough to start any extraction Affects extrinsic system and increases prothrombin time Heparin can be given subcutaneously and acts rapidly It takes at least 12 hours for Vitamin K to reverse the effects of coumarin 1029. Liver damage in mild overdose 1032. Extract the deciduous teeth C. space is sufficient. B. In regards to periapical lesions. Folate papillae Filiform papillae Neurofibroma Papilloma Collagen synthesis Clotting factor production Epithelial integrity Cellular energy production Topical fluorides are MOST beneficial when: A. HSV 1 B. Less than 35 degree Less than 45 degree Less than 90 degree Less than 100 degree 1036. C. Rheumatic arthritis C. D. Rheumatic fever B. 1035. C. D. The MOST PROBABLE diagnosis is: A. B. the MOST probable diagnosis is: A. Anterior inclined plane on mandibular teeth Bite plane Expansion screw Hawley appliance The angle of blade for closed curettage is:** A. Osteoarthritis . Patient complains of finger-like growth on the lateral aspect of the tongue. Applied after eruption 1040. Pemphigus 1037. In which of the following conditions vesicles/bullae are never seen prior to ulceration: A. D. 1039. B. Directly applied on decalcified enamel B. D.A. C. 1038. Aphthous ulcer C. B. Thiamine is useful in: A. Child presented to you with sore throat. B. The lesion is painless and of normal colour. C. fever and joint swelling. E. Allantoin 1042. What your next step would be: A. Uric acid C. 1044. D. C. 1046. Squamous cell carcinoma HIV gingivitis Osteosarcoma External lymphoma Kaposi sarcoma The MOST primary treatment of ANUG in HIV patient is: A. Perform an excision biopsy C. C. Garr’s osteomyelitis B. What would you do: . You “rounded off” the sharp area and recall patient after one month to see the lesion turning smaller in size. D. D. You want to place a post on an endodontically treated tooth which has a good silver point “Ag point”. B. B.1041. there is no evidence of failure of the previous root filing. Prescribe Kenalog and Orabase 1043. A patient on dicoumarol treatment needs extraction. C. Keep observing B. E. Patient shows a lesion on the tongue adjacent to sharp tooth. Torus 1047. Prescribe antibiotics Debridement and antimicrobial rinses Gingivoplasty Flap surgery 1045. End product of amino acids metabolism is: A. Urea B. Clotting time Bleeding time Prothrombin time Sedimentation rate Complete blood cell count Infection with new bone formation is: A. On HIV patient which of the following IS NOT RECOGNISED: A. B. Condensing osteitis C. Which of the following is MOST valuable in evaluating surgical risks: A. but the reason given is false 1050.A. Allergy because of denture C. Increase the compensatory curve B. The bilaminar zone in reference to TMJ refers to: A. Remove and replace the Ag point with Gutta Percha before the post preparation. When setting up teeth for complete dentures having bilateral balanced occlusion. what would you do to solve this: A. Upper buccal and lower lingual cusps 1055. The MOST common reason for full denture failure: A. Increasing the anterior posterior occlusal curve B. separation of posterior teeth during protrusion is done by: A. What is the MOST COMMON configuration of the mesial buccal canal of upper first molar:** . The upper and lower joint spaces B. In posterior crossbite situation which are the supporting cusps: A. Decreasing the angle orientation of the occlusal plane 1051. of: A complaint of burning tongue in an elderly female would be a result A. A systemic allergy B. The placement of metal stops at a location remote to direct retainers to increase retention is termed: A. Decrease the angle of the occlusal plane 1052. First statement is true. The hamular notch is important in full dentures construction because it aids in the setting position of the artificial teeth A. The distal attachments of the lateral pterygoid to the condyle 1056. 1048. Psychogenic 1054. Inadequate interocclusal clearance 1053. Indirect retainers 1049. When patient bites in protrusion you notice that posterior teeth do not meet. Tori A. Why would you invest the wax pattern as soon as possible in an indirect inlay fabrication: A. . Midline of the palate II. Two canals and one foramina 1057. None of the above E. Incisive foramen V. I and II B. II. II. Avoid contraction C. All of the above. What does “SYNERESIS” in prosthodontics mean: A. Loss of water and contraction 1058. I. Difficulties in keeping the intraoral pressure. Avoid expansion 1059. 1060.A. III C. Upon palpation which of the following areas would be found to have overlying mucosa: I. Mental foramen IV. Why do people with cleft palate/lip have speech difficulties: A. Mylohyoid ridge III. V D. I. Minimise distortion B.
Copyright © 2024 DOKUMEN.SITE Inc.